Синусы и косинусы формулы: тригонометрические формулы синус косинус суммы углов разности углов синус косинус двойного тройного углов синус косинус тангенс через тангенс половинного угла

Содержание

тригонометрические формулы синус косинус суммы углов разности углов синус косинус двойного тройного углов синус косинус тангенс через тангенс половинного угла



Справочник по математикеТригонометрия

Содержание

Связи между тригонометрическими функциями одного угла
Тригонометрические функции суммы и разности двух углов
Тригонометрические функции двойного угла
Формулы понижения степени для квадратов тригонометрических функций
Формулы понижения степени для кубов синуса и косинуса
Выражение тангенса угла через синус и косинус двойного угла
Преобразование суммы тригонометрических функций в произведение
Преобразование произведения тригонометрических функций в сумму
Выражение тригонометрических функций через тангенс половинного угла
Тригонометрические функции тройного угла

Связи между тригонометрическими функциями одного угла

sin2α + cos2α = 1

Тригонометрические функции суммы и разности двух углов

ФормулаНазвание формулы
sin (α + β) = sin α cos β + cos α sin βСинус суммы
sin (α – β) = sin α cos β – cos α sin βСинус разности
cos (α + β) = cos α cos β – sin α sin βКосинус суммы
cos (α – β) = cos α cos β + sin α sin β Косинус разности
Тангенс суммы
Тангенс разности
Синус суммы
sin (α + β) = sin α cos β +
+ cos α sin β
Синус разности
sin (α – β) = sin α cos β –
– cos α sin β
Косинус суммы
cos (α + β) = cos α cos β –
– sin α sin β
Косинус разности
cos (α – β) = cos α cos β +
+ sin α sin β
Тангенс суммы
Тангенс разности

Тригонометрические функции двойного угла

ФормулаНазвание формулы
sin 2α = 2 sin α cos αСинус двойного угла

cos 2α = cos 2α – sin2α

cos 2α = 2cos

2α – 1

cos 2α = 1 – 2sin 2α

Косинус двойного угла
Тангенс двойного угла
Синус двойного угла
sin 2α = 2 sin α cos α
Косинус двойного угла

cos 2α = cos 2α – sin2α

cos 2α = 2cos 2α – 1

cos 2α = 1 – 2sin 2α

Тангенс двойного угла

Формулы понижения степени для квадратов тригонометрических функций

ФормулаНазвание формулы

Выражение квадрата синуса через косинус двойного угла

Выражение квадрата косинуса через косинус двойного угла

Выражение квадрата тангенса через косинус двойного угла

Выражение квадрата синуса через косинус двойного угла
Выражение квадрата косинуса через косинус двойного угла
Выражение квадрата тангенса через косинус двойного угла

Формулы понижения степени для кубов синуса и косинуса

ФормулаНазвание формулы

Выражение куба синуса через синус угла и синус тройного угла

Выражение куба косинуса через косинус угла и косинус тройного угла

Выражение куба синуса через синус угла и синус тройного угла

Выражение куба косинуса через косинус угла и косинус тройного угла

Выражение тангенса через синус и косинус двойного угла

Преобразование суммы тригонометрических функций в произведение

ФормулаНазвание формулы
Сумма синусов
Разность синусов
Сумма косинусов
Разность косинусов
Сумма тангенсов
Разность тангенсов
Сумма синусов
Разность синусов
Сумма косинусов
Разность косинусов
Сумма тангенсов
Разность тангенсов

Преобразование произведения тригонометрических функций в сумму

ФормулаНазвание формулы
Произведение синусов
Произведение косинусов
Произведение синуса и косинуса
Произведение синусов
Произведение косинусов
Произведение синуса и косинуса

Выражение тригонометрических функций через тангенс половинного угла

ФормулаНазвание формулы

Выражение синуса угла через тангенс половинного угла

Выражение косинуса угла через тангенс половинного угла

Выражение тангенса угла через тангенс половинного угла

Выражение синуса угла через тангенс половинного угла
Выражение косинуса угла через тангенс половинного угла
Выражение тангенса угла через тангенс половинного угла

Тригонометрические функции тройного угла

ФормулаНазвание формулы
sin 3α = 3sin α – 4sin3αСинус тройного угла
cos 3α = 4cos3α –3cos αКосинус тройного угла
Тангенс тройного угла
Синус тройного угла
sin 3α = 3sin α – 4sin3α
Косинус тройного угла
cos 3α = 4cos3α –3cos α
Тангенс тройного угла

Наверх

Демонстрационные варианты ЕГЭ и ОГЭ

С демонстрационными вариантами ЕГЭ и ОГЭ по всем предметам, опубликованными на официальном информационном портале Единого Государственного Экзамена, можно ознакомиться на специальной страничке нашего сайта.

Наши учебные пособия для школьников

При подготовке к ЕГЭ и к ОГЭ по математике Вам могут также пригодиться наши учебные пособия.

  • Решение рациональных неравенств

  • Задачи на проценты

  • Решение показательных неравенств

  • Квадратный трехчлен

  • Метод координат на плоскости

  • Решение иррациональных неравенств

  • Фигуры на координатной плоскости

  • Решение алгебраических уравнений

  • Уравнения и неравенства с модулями

  • Решение показательных уравнений

  • Арифметическая и геометрическая прогрессии

  • Решение логарифмических уравнений

  • Решение логарифмических неравенств

  • Системы уравнений

  • Решение тригонометрических уравнений

  • Тригонометрия в ЕГЭ по математике

  • Степень с рациональным показателем

Формулы синусов и косинусов — онлайн справочник для студентов

Косинусы и синусы связаны между собою следующими тригонометрическими формулами. {2} \alpha=\frac{1+\cos 2 \alpha}{2} \)

Формулы для косинуса и синуса половинного аргумента \(\ \cos \frac{\alpha}{2}=\sqrt{\frac{1+\cos \alpha}{2}} \), \(\ \sin \frac{\alpha}{2}=\sqrt{\frac{1-\cos \alpha}{2}} \)

Формулы преобразования произведения косинусов и синусов в сумму

\(\ \sin \alpha \cdot \sin \beta=\frac{1}{2}[\cos (\alpha-\beta)-\cos (\alpha+\beta)] \)

\(\ \cos \alpha \cdot \cos \beta=\frac{1}{2}[\cos (\alpha+\beta)+\cos (\alpha-\beta)] \)

\(\ \sin \alpha \cdot \cos \beta=\frac{1}{2}[\sin (\alpha+\beta)+\sin (\alpha-\beta)] \)

Формулы преобразования суммы косинусов и синусов в произведение

\(\ \sin \alpha+\sin \beta=2 \sin \frac{\alpha+\beta}{2} \cdot \cos \frac{\alpha-\beta}{2} \)

\(\ \sin \alpha-\sin \beta=2 \sin \frac{\alpha-\beta}{2} \cdot \cos \frac{\alpha+\beta}{2} \)

\(\ \cos \alpha+\cos \beta=2 \cos \frac{\alpha+\beta}{2} \cdot \cos \frac{\alpha-\beta}{2} \)

\(\ \cos \alpha-\cos \beta=2 \sin \frac{\alpha+\beta}{2} \cdot \sin \frac{\beta-\alpha}{2} \)

Формулы, выражающие синус и косинус через тангенс половинного аргумента \(\ \sin \alpha=\frac{2 \operatorname{tg} \frac{\alpha}{2}}{1+\operatorname{tg}^{2} \frac{\alpha}{2}} \), \(\ \cos \alpha=\frac{1-\operatorname{tg}^{2} \frac{\alpha}{2}}{1+\operatorname{tg}^{2} \frac{\alpha}{2}} \)

Примеры решения задач

ПРИМЕР 1

  • Задание

    Доказать тождество \(\ \cos ^{2}(\alpha-\beta)-\cos ^{2}(\alpha+\beta)=\sin 2 \alpha \cdot \sin 2 \beta \)

  • Доказательство

    Распишем выражение в левой части заданного равенства как разность квадратов, получим:\(\ \cos ^{2}(\alpha-\beta)-\cos ^{2}(\alpha+\beta)=(\cos (\alpha-\beta)+\cos (\alpha+\beta)) \cdot(\cos (\alpha-\beta)-\cos (\alpha+\beta)) \)

    Далее преобразуем разность и сумму косинусов в скобках, используя формулы \(\ \cos \alpha+\cos \beta=2 \cos \frac{\alpha+\beta}{2} \cdot \cos \frac{\alpha-\beta}{2} ; \quad \cos \alpha-\cos \beta=2 \sin \frac{\alpha+\beta}{2} \cdot \sin \frac{\beta-\alpha}{2} \)

    Получим:

    \(\ \cos ^{2}(\alpha-\beta)-\cos ^{2}(\alpha+\beta)=2 \cos \frac{\alpha-\beta+\alpha+\beta}{2} \cdot \cos \frac{\alpha-\beta-\alpha-\beta}{2} \cdot 2 \sin \frac{\alpha-\beta+\alpha+\beta}{2} \cdot \sin \frac{\alpha+\beta-\alpha+\beta}{2}=2 \cos \alpha \cdot \cos (-\beta) \cdot 2 \sin \alpha \cdot \sin \beta \)

    Учитывая, что косинус функция четная, а также, используя формулу синуса двойного угла \(\ \sin 2 \alpha=2 \sin \alpha \cdot \cos \alpha \) , окончательно имеем: \(\ \cos ^{2}(\alpha-\beta)-\cos ^{2}(\alpha+\beta)=2 \sin \alpha \cdot \cos \alpha \cdot 2 \sin \beta \cdot \cos \beta=\sin 2 \alpha \cdot \sin 2 \beta \)

    Что и требовалось доказать. {4} \alpha \)

  • Физика

    166

    Реклама и PR

    31

    Педагогика

    80

    Психология

    72

    Социология

    7

    Астрономия

    9

    Биология

    30

    Культурология

    86

    Экология

    8

    Право и юриспруденция

    36

    Политология

    13

    Экономика

    49

    Финансы

    9

    История

    16

    Философия

    8

    Информатика

    20

    Право

    35

    Информационные технологии

    6

    Экономическая теория

    7

    Менеджент

    719

    Математика

    338

    Химия

    20

    Микро- и макроэкономика

    1

    Медицина

    5

    Государственное и муниципальное управление

    2

    География

    542

    Информационная безопасность

    2

    Аудит

    11

    Безопасность жизнедеятельности

    3

    Архитектура и строительство

    1

    Банковское дело

    1

    Рынок ценных бумаг

    6

    Менеджмент организации

    2

    Маркетинг

    238

    Кредит

    3

    Инвестиции

    2

    Журналистика

    1

    Конфликтология

    15

    Этика

    9

    Формулы дифференцирования Таблица синусов и косинусов Свойства тригонометрических функций Графики тригонометрических функций Тригонометрические неравенства и их решения

    Узнать цену работы

    Узнай цену

    своей работы

    Имя

    Выбрать тип работыЧасть дипломаДипломнаяКурсоваяКонтрольнаяРешение задачРефератНаучно — исследовательскаяОтчет по практикеОтветы на билетыТест/экзамен onlineМонографияЭссеДокладКомпьютерный набор текстаКомпьютерный чертежРецензияПереводРепетиторБизнес-планКонспектыПроверка качестваЭкзамен на сайтеАспирантский рефератМагистерскаяНаучная статьяНаучный трудТехническая редакция текстаЧертеж от рукиДиаграммы, таблицыПрезентация к защитеТезисный планРечь к дипломуДоработка заказа клиентаОтзыв на дипломПубликация в ВАКПубликация в ScopusДиплом MBAПовышение оригинальностиКопирайтингДругое

    Принимаю  Политику  конфиденциальности

    Подпишись на рассылку, чтобы не пропустить информацию об акциях

    Формулы (тождества) синус, косинус, тангенс, котангенс тройного угла


    Как найти,

    гипотенузу или катеты в прямоугольном треугольнике.

     

    a, b — катеты

    c — гипотенуза

    α, β — острые углы

     

    Формулы для катета, (a):

     

    Формулы для катета, (b):

     

     

    Формулы для гипотенузы, (c):

     

    Формулы сторон по теореме Пифагора, (a,b):

     




    Вычислить длину неизвестной стороны через любые стороны и углы

    b — сторона (основание)

    a — равные стороны

    α — углы при основании

    β — угол образованный равными сторонами

     

     

    Формулы длины стороны (основания), (b):

     

     

    Формулы длины равных сторон , (a):

     




    Вычислить длину стороны треугольника: по стороне и двум углам или по двум сторонам и углу.

     

     

    a, b, c — стороны произвольного треугольника

    α, β, γ — противоположные углы

     

     

    Формула  длины через две стороны и угол (по теореме косинусов), (a):

    * Внимательно, при подстановке в формулу, для тупого угла (α>90), cosα принимает отрицательное значение

     

     

    Формула  длины через сторону и два угла (по теореме синусов), (a):

     




    В прямоугольном треугольнике катеты, являются высотами. Ортоцентр — точка пересечения высот, совпадает с вершиной прямого угла.

     

    H — высота из прямого угла

    a, b — катеты

    с — гипотенуза

    c1 , c2 — отрезки полученные от деления гипотенузы, высотой

    α, β — углы при гипотенузе

     

    Формула длины высоты через стороны, (H):

     

     

    Формула длины высоты через гипотенузу и острые углы, (H):

     

    Формула длины высоты через катет и угол, (H):

     

    Формула длины высоты через составные отрезки гипотенузы , (H):




    Высота— перпендикуляр выходящий из любой вершины треугольника, к противоположной стороне (или ее продолжению, для треугольника с тупым углом).

    Высоты треугольника пересекаются в одной точке, которая называется — ортоцентр.

     

    H — высота треугольника

    a — сторона, основание

    b, c — стороны

    β, γ — углы при основании

    p — полупериметр, p=(a+b+c)/2

    R — радиус описанной окружности

    S — площадь треугольника

     

    Формула длины высоты через стороны, (H):

     

     

    Формула длины высоты через сторону и угол, (H):

     

    Формула длины высоты через сторону и площадь, (H):

     

    Формула длины высоты через стороны и радиус, (H):

     

     




    Медиана, отрезок |CO|, исходящий из вершины прямого угла BCA и делящий гипотенузу c, пополам.

    Медиана в прямоугольном треугольнике (M), равна, радиусу описанной окружности (R).

     

    M — медиана

    R — радиус описанной окружности

    O — центр описанной окружности

    с — гипотенуза

    a, b — катеты

    α — острый угол CAB

     

    Медиана равна радиусу и половине гипотенузы, (M):

     

     

    Формула длины через катеты, (M):

     

    Формула длины через катет и острый угол, (M):

     

     




    Медиана — отрезок |AO|, который выходит из вершины A и делит противолежащею сторону  c пополам.

    Медиана делит треугольник ABC на два равных по площади треугольника AOC и ABO.

     

     

    M — медиана, отрезок |AO|

    c — сторона на которую ложится медиана

    a, b — стороны треугольника

    γ — угол CAB

     

    Формула длины медианы через три стороны, (M):

     

     

    Формула длины медианы через две стороны и угол между ними, (M):




    Формула для вычисления высоты = биссектрисы = медианы.

    В равностороннем треугольнике: все высоты, биссектрисы и медианы, равны. Точка их пересечения, является центром вписанной окружности.

     

    L — высота=биссектриса=медиана

    a — сторона треугольника

     

     

    Формула длины высоты, биссектрисы и медианы равностороннего треугольника, (L):

     

     

    Калькулятор — вычислить, найти медиану, биссектрису, высоту

     




    Формулы для вычисления высоты, биссектрисы и медианы.

    В равнобедренном треугольнике: высота, биссектриса и медиана, исходящие из угла образованного равными сторонами, один и тот же отрезок.

     

    L — высота = биссектриса = медиана

    a — одинаковые стороны треугольника

    b — основание

    α — равные углы при основании

    β — угол образованный равными сторонами

     

    Формулы высоты, биссектрисы и медианы, через сторону и угол, (L):

     

     

     

     

    Формула высоты, биссектрисы и медианы, через стороны, (L):

     




    1. Найти по формулам длину биссектрисы из прямого угла на гипотенузу:

     

    L — биссектриса, отрезок ME ,  исходящий из прямого угла (90 град)

    a, b — катеты прямоугольного треугольника

    с — гипотенуза

    α — угол прилежащий к гипотенузе

     

    Формула длины биссектрисы через катеты, ( L):

     

    Формула длины биссектрисы через гипотенузу и угол, ( L):

     

     

    2. Найти по формулам длину биссектрисы из острого угла на катет:

     

    L — биссектриса, отрезок ME ,  исходящий из острого угла

    a, b — катеты прямоугольного треугольника

    с — гипотенуза

    α, β — углы прилежащие к гипотенузе

     

    Формулы длины биссектрисы через катет и угол, (L):

     

    Формула длины биссектрисы через катет и гипотенузу, (L):

     




    L— биссектриса, отрезок |OB|, который делит угол ABC пополам

    a, b — стороны треугольника

    с — сторона на которую опущена биссектриса

    d, e — отрезки полученные делением биссектрисы

    γ — угол ABC , разделенный биссектрисой пополам

    p — полупериметр, p=(a+b+c)/2

     

    Длина биссектрисы через две стороны и угол, (L):

     

    Длина биссектрисы через полупериметр и стороны, (L):

     

     

    Длина биссектрисы через три стороны, (L):

     

    Длина биссектрисы через стороны и отрезки d, e, (L):

     

     

     

    Точка пересечения всех трех биссектрис треугольника ABC, совпадает с центром О, вписанной окружности.



    Все формулы косинусов и синусов. Основные тригонометрические тождества, их формулировки и вывод

    Синус острого угла α прямоугольного треугольника – это отношение противолежащего катета к гипотенузе.
    Обозначается так: sin α.

    Косинус острого угла α прямоугольного треугольника – это отношение прилежащего катета к гипотенузе.
    Обозначается так: cos α.


    Тангенс
    острого угла α – это отношение противолежащего катета к прилежащему катету.
    Обозначается так: tg α.

    Котангенс острого угла α – это отношение прилежащего катета к противолежащему.
    Обозначается так: ctg α.

    Синус, косинус, тангенс и котангенс угла зависят только от величины угла.

    Правила:

    Основные тригонометрические тождества в прямоугольном треугольнике:

    (α – острый угол, противолежащий катету b и прилежащий к катету a . Сторона с – гипотенуза. β – второй острый угол).

    b
    sin α = —
    c

    sin 2 α + cos 2 α = 1

    a
    cos α = —
    c

    1
    1 + tg 2 α = —
    cos 2 α

    b
    tg α = —
    a

    1
    1 + ctg 2 α = —
    sin 2 α

    a
    ctg α = —
    b

    1 1
    1 + — = —
    tg 2 α sin 2 α

    sin α
    tg α = —
    cos α


    При возрастании острого угла
    sin α и tg α возрастают, а cos α убывает.


    Для любого острого угла α:

    sin (90° – α) = cos α

    cos (90° – α) = sin α

    Пример-пояснение :

    Пусть в прямоугольном треугольнике АВС
    АВ = 6,
    ВС = 3,
    угол А = 30º.

    Выясним синус угла А и косинус угла В.

    Решение .

    1) Сначала находим величину угла В. Тут все просто: так как в прямоугольном треугольнике сумма острых углов равна 90º, то угол В = 60º:

    В = 90º – 30º = 60º.

    2) Вычислим sin A. Мы знаем, что синус равен отношению противолежащего катета к гипотенузе. Для угла А противолежащим катетом является сторона ВС. Итак:

    BC 3 1
    sin A = — = — = —
    AB 6 2

    3) Теперь вычислим cos B. Мы знаем, что косинус равен отношению прилежащего катета к гипотенузе. Для угла В прилежащим катетом является все та же сторона ВС. Это значит, что нам снова надо разделить ВС на АВ – то есть совершить те же действия, что и при вычислении синуса угла А:

    BC 3 1
    cos B = — = — = —
    AB 6 2

    В итоге получается:
    sin A = cos B = 1/2.

    sin 30º = cos 60º = 1/2.

    Из этого следует, что в прямоугольном треугольнике синус одного острого угла равен косинусу другого острого угла – и наоборот. Именно это и означают наши две формулы:
    sin (90° – α) = cos α
    cos (90° – α) = sin α

    Убедимся в этом еще раз:

    1) Пусть α = 60º. Подставив значение α в формулу синуса, получим:
    sin (90º – 60º) = cos 60º.
    sin 30º = cos 60º.

    2) Пусть α = 30º. Подставив значение α в формулу косинуса, получим:
    cos (90° – 30º) = sin 30º.
    cos 60° = sin 30º.

    (Подробнее о тригонометрии — см.раздел Алгебра)

      Если известен угол треугольника, то можно воспользоваться специальным справочником и посмотреть там синус данного угла. Если же не известен угол, но то можно воспользоваться теоремой синусов. В частном случае, синус угла в прямоугольном треугольнике равен отношению противолежащего катета к гипотенузе.

      Давайте дадим определение, что же такое синус.

      Синус угла (sin) в треугольнике это отношение противолежащего катета к гипотенузе.

      Так что найти синус угла довольно таки просто, если есть значение катета и гипотенузы.

      Чтобы найти синус угла в любом треугольнике, необходимо воспользоваться формулами. Вот на этом рисунке показаны основные формулы, позволяющие рассчитывать синус угла в треугольнике:

      Воспользуйтесь этими формулами для рассчтеа.

      Если величина угла неизвестна, то так: синус угла равен отношению длины противолежащей рассматриваемому углу стороны к диаметру описанной вокруг треугольника окружности. А как найти этот диаметр? Нужно найти центр описанной окружности. Для этого через середины любых двух сторон треугольника провести перпендикуляры. Точка пересечения этих перпендикуляров и есть центр описанной окружности. Расстояние от нее до любой вершины треугольника — радиус описанной окружности.

      Чтобы ответить правильно на данный вопрос, нужно уточнить, синус угла в каком треугольнике нужно найти. Если этот треугольник произвольный , то это мы можем сделать только по теореме синусов (здесь см. исчерпывающий ответ Алекса).

      Если же нужно найти синус острого угла в прямоугольном треугольнике, то нужно воспользоваться определением синуса угла (как отношения противолежащего катета к гипотенузе). Тогда ответом будет: синус угла А = ВС/АВ, где ВС — противолежащий катет, АВ — гипотенуза.

      Доброго времени суток.

      Для нахождения синуса угла/углов прямоугольного треугольника можно воспользоваться двумя способами:

      • первый из них — это взять транспортир и найти угол треугольника (сколько градусов), а затем уже по таблице найти синус данного угла;
      • второй метод — это воспользоваться формулой нахождения синуса угла, который, как мы знаем, равен отношению противолежащего катета к гипотенузе.

      Можно найти синус угла двумя способами и сравнить значения.

      Все довольно просто.

      Я так понял, что задача сводится к тому, что нам неизвестен угол треугольника, и нам нужно его найти.

      Для того чтобы найти синус угла, а затем и сам угол в произвольном треугольнике, необходимо знать длины двух сторон: стороны, противолежащей искомому углу, и какой-либо другой стороны и ещ величину угла, противолежащего этой последней стороне.

      А затем нужно применить теорему синусов.

      Обозначим искомый (неизвестный) угол как A, противолежащую сторону a, другую известную сторону b, известный противолежащий этой стороне угол B.

      По теореме синусов: a/sin(A) = b/sin(B).

      Отсюда: sin(A) = a * sin(B)/b ;

      A = arcsina * sin(B)/b.

      В случае прямоугольного треугольника задача на нахождение синуса любого угла сводится всего лишь к вычислению отношения противолежащего от угла катета к гипотенузе — полученное значение и будет синусом. В произвольном треугольнике найти синус угла уже сложнее, но также возможно. Для этого надо хоть что-то знать из параметров треугольника. Например если известны три стороны треугольника, то углы находятся по теореме косинусов, а потом при желании легко находится синус уже найденного угла.

    В этой статье мы всесторонне рассмотрим . Основные тригонометрические тождества представляют собой равенства, устанавливающие связь между синусом, косинусом, тангенсом и котангенсом одного угла, и позволяют находить любую из этих тригонометрических функций через известную другую.

    Сразу перечислим основные тригонометрические тождества, которые разберем в этой статье. Запишем их в таблицу, а ниже дадим вывод этих формул и приведем необходимые пояснения.

    Навигация по странице.

    Связь между синусом и косинусом одного угла

    Иногда говорят не об основных тригонометрических тождествах, перечисленных в таблице выше, а об одном единственном основном тригонометрическом тождестве вида . Объяснение этому факту достаточно простое: равенства получаются из основного тригонометрического тождества после деления обеих его частей на и соответственно, а равенства и следуют из определений синуса, косинуса, тангенса и котангенса . Подробнее об этом поговорим в следующих пунктах.

    То есть, особый интерес представляет именно равенство , которому и дали название основного тригонометрического тождества.

    Прежде чем доказать основное тригонометрическое тождество, дадим его формулировку: сумма квадратов синуса и косинуса одного угла тождественно равна единице. Теперь докажем его.

    Основное тригонометрическое тождество очень часто используется при преобразовании тригонометрических выражений . Оно позволяет сумму квадратов синуса и косинуса одного угла заменять единицей. Не менее часто основное тригонометрическое тождество используется и в обратном порядке: единица заменяется суммой квадратов синуса и косинуса какого-либо угла.

    Тангенс и котангенс через синус и косинус

    Тождества, связывающие тангенс и котангенс с синусом и косинусом одного угла вида и сразу следуют из определений синуса, косинуса, тангенса и котангенса. Действительно, по определению синус есть ордината y, косинус есть абсцисса x, тангенс есть отношение ординаты к абсциссе, то есть, , а котангенс есть отношение абсциссы к ординате, то есть, .

    Благодаря такой очевидности тождеств и часто определения тангенса и котангенса дают не через отношение абсциссы и ординаты, а через отношение синуса и косинуса. Так тангенсом угла называют отношение синуса к косинусу этого угла, а котангенсом – отношение косинуса к синусу.

    В заключение этого пункта следует отметить, что тождества и имеют место для всех таких углов , при которых входящие в них тригонометрические функции имеют смысл. Так формула справедлива для любых , отличных от (иначе в знаменателе будет нуль, а деление на нуль мы не определяли), а формула — для всех , отличных от , где z — любое .

    Связь между тангенсом и котангенсом

    Еще более очевидным тригонометрическим тождеством, чем два предыдущих, является тождество, связывающее тангенс и котангенс одного угла вида . Понятно, что оно имеет место для любых углов , отличных от , в противном случае либо тангенс, либо котангенс не определены.

    Доказательство формулы очень просто. По определению и , откуда . Можно было доказательство провести и немного иначе. Так как и , то .

    Итак, тангенс и котангенс одного угла, при котором они имеют смысл, есть .

    Учителя считают, что каждый школьник должен уметь проводить расчёты, знать тригонометрические формулы, но далеко не каждый преподаватель объясняет, что такое синус и косинус. Каков их смысл, где они используются? Почему мы говорим про треугольники, а в учебнике нарисована окружность? Попробуем связать все факты воедино.

    Школьный предмет

    Изучение тригонометрии начинается обычно в 7-8 классе средней школы. В это время учащимся объясняют, что такое синус и косинус, предлагают решать геометрические задачи с применением этих функций. Позже появляются более сложные формулы и выражения, которые требуется алгебраическим способом преобразовывать (формулы двойного и половинного угла, степенные функции), проводится работа с тригонометрической окружностью.

    Однако учителя далеко не всегда могут доходчиво объяснить смысл используемых понятий и применимость формул. Поэтому ученик зачастую не видит смысла в данном предмете, а заученная информация быстро забывается. Однако стоит один раз объяснить старшекласснику, например, связь между функцией и колебательным движением, и логическая связь запомнится на многие годы, а шутки на тему бесполезности предмета уйдут в прошлое.

    Использование

    Заглянем ради любопытства в различные разделы физики. Хотите определить дальность полёта снаряда? Или высчитываете силу трения между объектом и некой поверхностью? Раскачиваете маятник, следите за лучами, проходящими сквозь стекло, высчитываете индукцию? Практически в любой формуле фигурируют тригонометрические понятия. Так что такое синус и косинус?

    Определения

    Синус угла представляет собой отношение противолежащего катета к гипотенузе, косинус — прилежащего катета всё к той же гипотенузе. Здесь нет совершенно ничего сложного. Возможно, учеников обычно смущают значения, которые они видят в тригонометрической таблице, ведь там фигурируют квадратные корни. Да, получать из них десятичные дроби не очень удобно, но кто сказал, что все числа в математике должны быть ровными?

    На самом деле в задачниках по тригонометрии можно найти забавную подсказку: большинство ответов здесь ровные и в худшем случае содержат корень из двух или из трёх. Вывод прост: если у вас в ответе получилась «многоэтажная» дробь, перепроверьте решение на предмет ошибок в расчётах или в рассуждениях. И вы их, скорее всего, найдете.

    Что нужно запомнить

    Как и в любой науке, в тригонометрии есть такие данные, которые необходимо выучить.

    Во-первых, следует запомнить числовые значения для синусов, косинусов прямоугольного треугольника 0 и 90, а также 30, 45 и 60 градусов. Эти показатели встречаются в девяти из десяти школьных задач. Подглядывая эти значения в учебнике, вы потеряете много времени, а на контрольной или экзамене посмотреть и вовсе будет негде.

    Нужно помнить, что значение обеих функций не может превышать единицу. Если где-либо в расчетах вы получите значение, выходящее за пределы диапазона 0-1, остановитесь и решите задачу заново.

    Сумма квадратов синуса и косинуса равна единице. Если вы уже нашли одно из значений, воспользуйтесь этой формулой для нахождения оставшегося.

    Теоремы

    В базовой тригонометрии существует две основные теоремы: синусов и косинусов.

    Первая гласит, что отношение каждой стороны треугольника к синусу противолежащего угла одинаково. Вторая — что квадрат любой стороны можно получить, если сложить квадраты двух оставшихся сторон и вычесть удвоенное их произведение, умноженное на косинус лежащего между ними угла.

    Таким образом, если в теорему косинусов подставить значение угла в 90 градусов, мы получим… теорему Пифагора. Теперь, если требуется высчитать площадь фигуры, не являющейся прямоугольным треугольником, можно больше не переживать — две рассмотренные теоремы существенно упростят решение задачи.

    Цели и задачи

    Изучение тригонометрии значительно упростится, когда вы осознаете один простой факт: все выполняемые вами действия направлены на достижения всего одной цели. Любые параметры треугольника могут быть найдены, если вы знаете о нём самый минимум информации — это может быть величина одного угла и длины двух сторон или, например, три стороны.

    Для определения синуса, косинуса, тангенса любого угла этих данных достаточно, с их же помощью можно легко высчитать площадь фигуры. Практически всегда в качестве ответа требуется привести одно из упомянутых значений, а найти их можно по одним и тем же формулам.

    Нестыковки при изучении тригонометрии

    Одним из непонятных вопросов, которых школьники предпочитают избегать, является обнаружение связи между различными понятиями в тригонометрии. Казалось бы, для изучения синусов и косинусов углов используются треугольники, но обозначения почему-то часто встречаются на рисунке с окружностью. Кроме того, существует и вовсе непонятный волнообразный график под названием синусоида, не имеющий никакого внешнего сходства ни с окружностью, ни с треугольниками.

    Более того, углы измеряются то в градусах, то в радианах, а число Пи, записывающееся просто как 3,14 (без единиц измерения), почему-то фигурирует в формулах, соответствуя 180 градусам. Как всё это связано между собой?

    Единицы измерения

    Почему число Пи равняется именно 3,14? Помните ли вы, что это за значение? Это количество радиусов, умещающихся в дуге на половине окружности. Если диаметр круга — 2 сантиметра, длина окружности составит 3,14*2, или 6,28.

    Второй момент: возможно, вы замечали сходство слов «радиан» и «радиус». Дело в том, что один радиан численно равен величине угла, отложенного из центра окружности на дугу длиной в один радиус.

    Теперь совместим полученные знания и поймем, почему сверху на оси координат в тригонометрии пишется «Пи пополам», а слева — «Пи». Это угловая величина, измеренная в радианах, ведь полукруг — это 180 градусов, или 3,14 радиана. А там, где есть градусы, есть синусы и косинусы. Треугольник же легко провести от нужной точки, отложив отрезки к центру и на ось координат.

    Заглянем в будущее

    Тригонометрия, изучаемая в школе, имеет дело с прямолинейной системой координат, где, как бы это странно ни звучало, прямая — это прямая.

    Но есть и более сложные способы работы с пространством: сумма углов треугольника здесь будет больше 180 градусов, а прямая в нашем представлении будет выглядеть как самая настоящая дуга.

    Перейдем от слов к делу! Возьмите яблоко. Сделайте ножом три надреза, чтобы при взгляде сверху получался треугольник. Выньте получившийся кусок яблока и посмотрите на «рёбра», где заканчивается кожура. Они вовсе не прямые. Фрукт в ваших руках условно можно назвать круглым, а теперь представьте, какими сложными должны быть формулы, с помощью которых можно найти площадь вырезанного куска. А ведь некоторые специалисты решают такие задачи ежедневно.

    Тригонометрические функции в жизни

    Обращали ли вы внимание, что самый короткий маршрут самолёта из точки А в точку Б на поверхности нашей планеты имеет ярко выраженную форму дуги? Причина проста: Земля имеет форму шара, а значит, с помощью треугольников многого не вычислишь — здесь приходится использовать более сложные формулы.

    Не обойтись без синуса/косинуса острого угла в любых вопросах, связанных с космосом. Интересно, что здесь сходится целое множество факторов: тригонометрические функции требуются при расчётах движения планет по окружностям, эллипсам и различным траекториям более сложных форм; процесса запуска ракет, спутников, шаттлов, отстыковки исследовательских аппаратов; наблюдении за далёкими звёздами и изучении галактик, до которых человек в обозримом будущем добраться не сможет.

    В целом поле для деятельности человека, владеющего тригонометрией, очень широко и, по-видимому, со временем будет только расширяться.

    Заключение

    Сегодня мы узнали или, во всяком случае, повторили, что такое синус и косинус. Это понятия, которых не нужно бояться — стоит захотеть, и вы поймете их смысл. Помните, что тригонометрия — это не цель, а лишь инструмент, который можно использовать для удовлетворения реальных человеческих потребностей: строить дома, обеспечивать безопасность движения, даже осваивать просторы вселенной.

    Действительно, сама по себе наука может казаться скучной, но как только вы найдете в ней способ достижения собственных целей, самореализации, процесс обучения станет интересным, а ваша личная мотивация возрастёт.

    В качестве домашнего задания попробуйте найти способы применить тригонометрические функции в той сфере деятельности, которая интересна лично вам. Пофантазируйте, включите воображение, и тогда наверняка окажется, что новые знания пригодятся вам в будущем. Да и кроме того, математика полезна для общего развития мышления.

    Инструкция

    Используйте функцию арксинус для вычисления величины угла в градусах, если известно значение этого угла. Если угол обозначить буквой α, в общем виде решение можно записать так: α = arcsin(sin(α)).

    Если у вас есть возможность пользоваться компьютером, для практических расчетов проще всего использовать встроенный операционной системы. В последних двух версиях ОС Windows его можно запустить так: нажмите клавишу Win, наберите «ка» и надавите Enter. В более ранних выпусках этой ОС ссылку «Калькулятор» ищите в подразделе «Стандартные» раздела «Все программы» главного меню системы.

    После запуска приложения переключите его в режим, позволяющий работать с тригонометрическими функциями. Сделать это можно выбором строки «Инженерный» в разделе «Вид» меню калькулятора или нажатием клавиш Alt + 2.

    Введите значение синуса. По умолчанию в интерфейсе калькулятора нет кнопки для вычисления арксинуса. Чтобы возможность использовать эту функцию, вам нужно инвертировать значения кнопок по умолчанию — кликните по клавише Inv в окне программы. В более ранних версиях эту кнопку заменяет чекбокс с таким же обозначением — поставьте в нем отметку.

    Можно использовать в расчетах и различные -сервисы, которых более чем достаточно в интернете. Например, перейдите на страницу http://planetcalc.com/326/, прокрутите ее немного вниз и в поле Input введите значение синуса. Для запуска процедуры вычисления здесь предназначена кнопка с надписью Calculate — кликните по ней. Результат вычислений вы найдете в первой строке таблицы под этой кнопкой. Кроме арксинуса в ней отображаются и величины , и арккотангенса введенного значения.

    Обратная синусу тригонометрическая функция называется арксинусом . Она может принимать значения, лежащие в пределах половины числа Пи как в положительную, так и в отрицательную стороны при измерении в радианах. При измерении в градусах эти значения будут находиться, соответственно, в диапазоне от -90° до +90°.

    Инструкция

    Некоторые «круглые» значения не обязательно вычислять, проще их запомнить. Например:- если аргумент функции равен нулю, то значение арксинуса от него тоже равно нулю;- от 1/2 равен 30° или 1/6 Пи, если измерять ;- арксинус от -1/2 равен -30° или -1/6 от числа Пи в ;- арксинус от 1 равен 90° или 1/2 от числа Пи в радианах;- арксинус от -1 равен -90° или -1/2 от числа Пи в радианах;

    Для измерения значений этой функции от других аргументов проще всего воспользоваться стандартным калькулятором Windows, если под рукой есть . Чтобы запустить раскройте главное меню на кнопке «Пуск» ( или нажатием клавиши WIN), перейдите в раздел «Все программы», а затем в подраздел «Стандартные» и щелкните пункт «Калькулятор».

    Переключите интерфейс калькулятора в тот режим работы, который позволяет вычислять тригонометрические функции. Для этого откройте в его меню раздел «Вид» и выберите пункт «Инженерный» или «Научный» (в зависимости от используемой операционной системы).

    Введите значение аргумента, от которого надо вычислить арктангенс. Это можно делать, щелкая кнопки интерфейса калькулятора мышкой, или нажимая клавиши на , или скопировав значение (CTRL + C) и затем вставив его (CTRL + V) в поле ввода калькулятора.

    Выберите единицы измерения, в которых вам нужно получить результат вычисления функции. Ниже поля ввода помещены три варианта, из которых вам нужно выбрать (щелкнув его мышкой) одни — , радианы или рады.

    Поставьте отметку в чекбоксе, который инвертирует функции, указанные на кнопках интерфейса калькулятора. Рядом с ним стоит короткая надпись Inv.

    Щелкните кнопку sin. Калькулятор инвертирует привязанную к ней функцию, произведет вычисление и представит вам результат в заданных единицах измерения.

    Видео по теме

    На прямоугольном треугольнике, как наипростейшем из многоугольников, разные ученые мужи оттачивали свои знания в области тригонометрии еще в те времена, когда эту область математики никто даже не называл таким словом. Поэтому указать автора, выявившего закономерности в соотношениях длин сторон и величин углов в этой плоской геометрической фигуре, сегодня не представляется возможным. Такие соотношения названы тригонометрическими функциями и поделены на несколько групп, основной из которых условно считаются «прямые» функции. К этой группе отнесены всего две функции и одна из них — синус.

    Инструкция

    По определению в прямоугольном треугольнике один из углов равен 90°, а в силу того, что сумма его углов в евклидовой геометрии обязана быть равной 180°, два других угла являются (т. е. 90°). Закономерности соотношений именно этих углов и длин сторон и описывают тригонометрические функции.

    Функция, называемая синусом острого угла, определяет соотношение между длиной двух сторон прямоугольного треугольника, одна из которых лежит напротив этого острого угла, а другая примыкает к нему и лежит напротив прямого угла. Так как сторона, лежащая напротив прямого угла в таком треугольнике, называется гипотенузой, а две другие — катетами, то функции синус можно сформулировать как соотношение между длинами катета и гипотенузы.

    Кроме такого простейшего определения этой тригонометрической функции и более сложные: через окружность в декартовых координатах, через ряды, через дифференциальных и функциональных уравнений. Эта функция непрерывна, то есть ее аргументами («областью определений») может быть любое число — от бесконечно отрицательного до бесконечно положительного. А максимум значений этой функции ограничены диапазоном от -1 до +1 — это «область ее значений». Минимальное значение синус принимает при угле в 270°, что соответствует 3/ Пи, а максимальное получается при 90° (½ от Пи). Нулевыми значения функции становятся при 0°, 180°, 360° и т.д. Из всего этого вытекает, что синус является функцией периодической и период ее равен 360° или удвоенному числу Пи.

    Для практических расчетов значений этой функции от заданного аргумента можно использовать — абсолютное большинство из них (включая программный калькулятор, встроенный в операционную систему вашего компьютера) имеет соответствующую опцию.

    Видео по теме

    Синус и косинус — это прямые тригонометрические функции, для которых существует несколько определений — через окружность в декартовой системе координат, через решения дифференциального уравнения, через острые углы в прямоугольном треугольнике. Каждое из таких определений позволяет вывести зависимость между этими двумя функциями. Ниже приведен самый, пожалуй, простой способ выразить косинус через синус — через их определения для острых углов прямоугольного треугольника.

    Инструкция

    Выразите синус острого угла прямоугольного треугольника через длины сторон этой фигуры. Согласно определению, синус угла (α) должен быть отношению длины стороны (a), лежащей напротив него — катета — к длине стороны (c), противолежащей прямому углу — гипотенузы: sin(α) = a/c.

    Найдите аналогичную формулу для косинус а того же угла. По определению эта величина должна выражаться отношением длины стороны (b), примыкающей к этому углу (второго катета), к длине стороны (c), лежащей напротив прямого угла: cos(а) = a/c.

    Перепишите равенство, вытекающее из теоремы Пифагора, таким образом, чтобы в нем были задействованы соотношения между катетами и гипотенузой, выведенные на двух предыдущих шагах. Для этого сначала разделите обе исходного этой теоремы (a² + b² = c²) на квадрат гипотенузы (a²/c² + b²/c² = 1), а затем полученное равенство перепишите в таком виде: (a/c)² + (b/c)² = 1.

    Замените в полученном выражении соотношения длин катетов и гипотенузы тригонометрическими функциями, исходя из формул первого и второго шага: sin²(а) + cos²(а) = 1. Выразите косинус из полученного равенства: cos(a) = √(1 — sin²(а)). На этом задачу можно решенной в общем виде.

    Если кроме общего нужно получить численный результат, воспользуйтесь, например, калькулятором, встроенным в операционную систему Windows. Ссылку на его запуск в подразделе «Стандартные» раздела «Все программы» меню ОС. Эта ссылка сформулирована лаконично — «Калькулятор». Чтобы иметь возможность вычислять с этой программы тригонометрические функции включите ее «инженерный» интерфейс — нажмите комбинацию клавиш Alt + 2.

    Введите в условиях значение синуса угла и кликните по кнопке интерфейса с обозначением x² — так вы возведете исходное значение в квадрат. Затем наберите на клавиатуре *-1, нажмите Enter, введите +1 и нажмите Enter еще раз — таким способом вы вычтите из единицы квадрат синуса. Щелкните по клавише со значком радикала, чтобы извлечь квадратный и получить окончательный результат.

    Изучение треугольников ведется математиками на протяжении нескольких тысячелетий. Наука о треугольниках — тригонометрия — использует специальные величины: синус и косинус.

    Прямоугольный треугольник

    Изначально синус и косинус возникли из-за необходимости рассчитывать величины в прямоугольных треугольниках. Было замечено, что если значение градусной меры углов в прямоугольном треугольнике не менять, то соотношение сторон, насколько бы эти стороны ни изменялись в длине, остается всегда одинаковым.

    Именно так и были введены понятия синуса и косинуса. Синус острого угла в прямоугольном треугольнике – это отношение противолежащего катета к гипотенузе, а косинус – прилежащего к гипотенузе.

    Теоремы косинусов и синусов

    Но косинусы и синусы могут применяться не только в прямоугольных треугольниках. Чтобы найти значение тупого или острого угла, стороны любого треугольника, достаточно применить теорему косинусов и синусов.

    Теорема косинусов довольно проста: «Квадрат стороны треугольника равен сумме квадратов двух других сторон за вычетом удвоенного произведения этих сторон на косинус угла между ними».

    Существует две трактовки теоремы синусов: малая и расширенная. Согласно малой: «В треугольнике углы пропорциональны противолежащим сторонам». Данную теорему часто расширяют за счет свойства описанной около треугольника окружности: «В треугольнике углы пропорциональны противолежащим сторонам, а их отношение равно диаметру описанной окружности».

    Производные

    Производная — математический инструмент, показывающий, как быстро меняется функция относительно изменения ее аргумента. Производные используются , геометрии, и , ряде технических дисциплин.

    При решении задач требуется знать табличные значения производных тригонометрических функций: синуса и косинуса. Производной синуса является косинус, а косинуса — синус, но со знаком «минус».

    Применение в математике

    Особенно часто синусы и косинусы используются при решении прямоугольных треугольников и задач, связанных с ними.

    Удобство синусов и косинусов нашло свое отражение и в технике. Углы и стороны было просто оценивать по теоремам косинусов и синусов, разбивая сложные фигуры и объекты на «простые» треугольники. Инженеры и , часто имеющие дело с расчетами соотношения сторон и градусных мер, тратили немало времени и усилий для вычисления косинусов и синусов не табличных углов.

    Тогда «на подмогу» пришли таблицы Брадиса, содержащие тысячи значений синусов, косинусов, тангенсов и котангенсов разных углов. В советское время некоторые преподаватели заставляли своих подопечных страницы таблиц Брадиса наизусть.

    Радиан — угловая величина дуги, по длине равной радиусу или 57,295779513° градусов.

    Градус (в геометрии) — 1/360-я часть окружности или 1/90-я часть прямого угла.

    π = 3.141592653589793238462… (приблизительное значение числа Пи).

    Таблица косинусов для углов: 0°, 30°, 45°, 60°, 90°, 120°, 135°, 150°, 180°, 210°, 225°, 240°, 270°, 300°, 315°, 330°, 360°.

    Угол х (в градусах)30°45°60°90°120°135°150°180°210°225°240°270°300°315°330°360°
    Угол х (в радианах)0 π/6π/4π/3π/22 x π/33 x π/45 x π/6π7 x π/65 x π/44 x π/33 x π/25 x π/37 x π/411 x π/62 x π
    cos x1 √3/2 (0,8660) √2/2 (0,7071) 1/2 (0,5) 0 -1/2 (-0,5) -√2/2 (-0,7071) -√3/2 (-0,8660) -1 -√3/2 (-0,8660) -√2/2 (-0,7071) -1/2 (-0,5) 0 1/2 (0,5) √2/2 (0,7071) √3/2 (0,8660) 1

    Урок 32.

    зависимость между синусом, косинусом и тангенсом одного и того же угла — Алгебра и начала математического анализа — 10 класс

    Алгебра и математического начала анализа, 10 класс

    Урок №32. Зависимость между синусом, косинусом и тангенсом одного и того же угла.

    Перечень вопросов, рассматриваемых в теме:

    • зависимость между синусом, косинусом и тангенсом одного и того же угла;
    • доказательство тригонометрических тождеств на основе зависимости между синусом, косинусом и тангенсом одного и того же угла;
    • решение несложных уравнений с использованием зависимости между синусом, косинусом и тангенсом одного и того же угла.
    • Упрощение тригонометрических выражений на основе зависимости между синусом, косинусом и тангенсом одного и того же угла.

    Глоссарий по теме

    Тождество — это равенство, верное для всех допустимых значений входящих в него букв (таких, при которых его левая и правая части имеют смысл, а задачи на доказательство таких равенств называют задачами на доказательство тождеств.

    Основная литература:

    Колягин Ю.М., Ткачева М.В, Федорова Н.Е. и др., под ред. Жижченко А.Б. Алгебра и начала математического анализа (базовый и профильный уровни) 10 кл.– М.: Просвещение, 2014.

    Открытые электронные ресурсы:

    Решу ЕГЭ образовательный портал для подготовки к экзаменам https://ege.sdamgia.ru/

    Теоретический материал для самостоятельного изучения

    Рассмотрим точку В(х;у), лежащую на тригонометрической окружности . Она получена поворотом точки А(1;0) вокруг начала координат на угол .

    Синусом угла является ордината точки В(х;у). Косинусом угла является её абсцисса.

    Рисунок 1 – точка В на тригонометрической окружности

    Образовался прямоугольный треугольник ОВС. По теореме Пифагора

    Катет ОС — это абсцисса точки В или , катет ВС- её ордината, или а гипотенуза ОВ — радиус единичной окружности, ОВ=1.Получаем формулу:

    (1)

    В тригонометрии её называют основным тригонометрическим тождеством. Она связывает синус с косинусом. А это значит, чо зная значения синуса, можно найти значения косинуса и наоборот.

    (2)

    (3)

    В этих равенствах знаки перед корнем определяются по знакам синуса и косинуса.

    Пример. Найти , если , .

    Выясним знак косинуса. Из условия опрелеляем, что угол в 4 четверти,

    Подставим значение в формулу (3), получаем:

    Ответ: .

    Пример. Могут ли одновременно выполняться равенства и

    Чтобы одновременно выполнялись эти равенства, необходимо выполнение условия

    . Подставим данные значения в формулу и проверим верно ли равенство: .

    ;

    ;

    1=1, верно.

    Ответ: данные равенства могут выполняться одновременно.

    Пример. Известно, что , найти .

    Возведём в квадрат левую и правую части равенста:

    ; учтём, что ,

    ;

    ;

    .

    А какая же зависимость между тангенсом и котангенсом одного угла?

    По определению : , .

    Перемножим эти равенства и получим формулу, которая связывает тангенс и котангенс:

    .

    , (4)

    и ,

    причём угол и

    Из этих формул видно, что тангенс и котангенс являются взаимнообратными числами.

    Если , то .

    Пример. Могут ли одновременно выполняться равенства и ? Подставляем данные значения в формулу (4) и получаем верное равенство.

    .

    Ответ: данные равенства могут выполняться одновременно.

    А есть ли связь между тангенсом и косинусом? Рассмотрим равенство

    и обе части возведём в квадрат:. Используя формулы (2) и (3), получаем:

    ,

    , (5)

    где

    По этой формуле можно находить значение тангенса по заданному значению косинуса и наоборот находить косинус, если известен тангенс.

    Пример . Известно, что ; . Найти , и .

    Угол в первой четверти, значит все значения положительны. Найдём их по тригонометрическим формулам.

    1. ;
    2. ;
    3. .

    Применяя тригонометрические формулы, можно зная одно из чисел , , и , найти остальные три. Эти формулы являются тождествами.

    Определение

    Равенство, верное для всех допустимых значений входящих в него букв (таких, при которых его левая и правая части имеют смысл), называется тождеством, а задачи на доказательство таких равенств называют задачами на доказательство тождеств.

    Рассмотрим некоторые приемы

    1. Левую часть приводят к правой, или наоборот правую к левой.
    2. Устанавливают то, что разность левой и правой частей равна нулю.

    Пример. Доказать тождество:

    Преобразуем левую часть:

    Левая часть тождества равна правой. Доказано.

    Примеры и разбор решения заданий тренировочного модуля

    Пример 1.

    Найти , если , .

    Из условия видим, что угол в 3 четверти, значит . Используем формулу (2):

    Ответ: .

    Пример 2.

    Найти , если , .

    Угол находится в 4 четверти, тангенс отрицательный. Подставим данное значение косинуса в формулу (5) и вычислим значение тангенса.

    .

    Ответ: .

    Пример 3.

    Доказать тождество:

    Преобразуем правую часть:

    Правая часть тождества равна левой. Доказано.

    что такое? Как найти синус, косинус и тангенс

    Косинус суммы и разности двух углов

    В этом параграфе будут доказаны следующие две формулы:

    cos (α + β) = cos α cos β — sin α sin β, (1)

    cos (α — β) = cos α cos β + sin α sin β. (2)

    Косинус суммы (разности) двух углов равен произведению косинусов этих углов минус (плюс) произведение синусов этих углов.

    Нам удобнее будет начать с доказательства формулы (2). Для простоты изложения предположим сначала, что углы α и β удовлетворяют следующим условиям:

    1) каждый из этих углов неотрицателен и меньше :

    0 α 2π, 0 β

    2) α > β .

    Пусть положительная часть оси 0х является общей начальной стороной углов α и β .

    Конечные стороны этих углов обозначим соответственно через 0А и 0В. Очевидно, что угол α β можно рассматривать как такой угол, на который нужно повернуть луч 0В вокруг точки 0 против часовой стрелки, чтобы его направление совпало с направлением луча 0А.

    На лучах 0А и 0В отметим точки М и N, отстоящие от начала координат 0 на расстоянии 1, так что 0М = 0N = 1.

    В системе координат х0у точка М имеет координаты (cos α, sin α ), а точка N — координаты (cos β , sin β ). Поэтому квадрат расстояния между ними равен:

    d 1 2 = (cos α — cos β) 2 + (sin α — sin β) 2 = cos 2 α — 2 cos α cos β +

    + cos 2 β + sin 2 α — 2sin α sin β + sin 2 β = .

    При вычислениях мы воспользовались тождеством

    sin 2 φ + cos 2 φ = 1 .

    Теперь рассмотрим другую систему координат В0С, которая получается путем поворота осей 0х и 0у вокруг точки 0 против часовой стрелки на угол β .

    В этой системе координат точка М имеет координаты (cos (α β ), sin (α β )), а точка N -координаты (1,0). Поэтому квадрат расстояния между ними равен:

    d 2 2 = 2 + 2 = cos 2 (α — β) — 2 cos (α — β) + 1 +

    + sin 2 (α — β) = 2 .

    Но расстояние между точками М и N не зависит от того, относительно какой системы координат мы рассматриваем эти точки. Поэтому

    d 1 2 = d 2 2

    2 (1 — cos α cos β — sin α sin β) = 2 .

    Отсюда и вытекает формула (2).

    Теперь следует вспомнить о тех двух ограничениях, которые мы наложили для простоты изложения на углы α и β .

    Требование, чтобы каждый из углов α и β был неотрицательным, на самом деле не существенно. Ведь к любому из этих углов можно прибавить угол, кратный 2я, что никак не отразится на справедливости формулы (2). Точно так же от каждого из данных углов можно вычесть угол, кратный . Поэтому можно считать, что 0 α 2π , 0 β 2π .

    Не существенным оказывается и условие α > β . Действительно, если α β , то β >α ; поэтому, учитывая четность функции cos х , получаем:

    cos (α — β) = cos (β — α) = cos β cos α + sin β sin α,

    что по существу совпадает с формулой (2). Таким образом, формула

    cos (α — β) = cos α cos β + sin α sin β

    верна для любых углов α и β . В частности, заменяя в ней β на —β и учитывая, что функция cos х является четной, а функция sin х нечетной, получаем:

    cos (α + β) = cos [α — (- β)] =cos α cos (-β) + sin α sin (-β) =

    = cos α cos β — sin α sin β,

    что доказывает формулу (1).

    Итак, формулы (1) и (2) доказаны.

    Примеры.

    1) cos 75° = cos (30° + 45°) = cos 30° cos 45°-sin 30°-sin 45° =

    2) cos 15° = cos (45° — 30°) = cos 45° cos 30° + sin 45° sin 30° =

    Упражнения

    1 . Вычислить, не пользуясь тригонометрическими таблицами:

    a) cos 17° cos 43° — sin 17° sin 43°;

    б) sin 3° sin 42° — cos 39° cos 42°;

    в) cos 29° cos 74° + sin 29° sin 74°;

    г) sin 97° sin 37° + cos 37° cos 97°;

    д) cos 3π / 8 cos π / 8 + sin 3π / 8 sin π / 8 ;

    e) sin 3π / 5 sin 7π / 5 — cos 3π / 5 cos 7π / 5 .

    2.Упростить выражения:

    a). cos (α + π / 3 ) + cos (π / 3 — α ) .

    б). cos (36° + α ) cos (24° — α ) + sin (36° + α ) sin (α — 24°).

    в). sin (π / 4 — α ) sin (π / 4 + α ) — cos (π / 4 + α ) cos (π / 4 — α )

    г) cos 2α + tg α sin 2α .

    3 . Вычислить :

    a) cos (α — β) , если

    cos α = — 2 / 5 , sin β = — 5 / 13 ;

    90° α β

    б) cos (α + π / 6), если cos α = 0,6;

    3π / 2 α

    4 . Найти cos (α + β) и cos (α — β) ,если известно, что sin α = 7 / 25 , cos β = — 5 / 13 и оба угла (α и β ) оканчиваются в одной и той же четверти.

    5 .Вычислить:

    а). cos [ arcsin 1 / 3 + arccos 2 / 3 ]

    б). cos [ arcsin 1 / 3 — arccos (- 2 / 3)] .

    в). cos [ arctg 1 / 2 + arccos (- 2) ]

    Одним из разделов математики, с которыми школьники справляются с наибольшими трудностями, является тригонометрия. Неудивительно: для того чтобы свободно овладеть этой областью знаний, требуется наличие пространственного мышления, умение находить синусы, косинусы, тангенсы, котангенсы по формулам, упрощать выражения, уметь применять в вычислениях число пи. Помимо этого, нужно уметь применять тригонометрию при доказательстве теорем, а это требует либо развитой математической памяти, либо умения выводить непростые логические цепочки.

    Истоки тригонометрии

    Знакомство с данной наукой следует начать с определения синуса, косинуса и тангенса угла, однако прежде необходимо разобраться, чем вообще занимается тригонометрия.

    Исторически главным объектом исследования данного раздела математической науки были прямоугольные треугольники. Наличие угла в 90 градусов дает возможность осуществлять различные операции, позволяющие по двум сторонам и одному углу либо по двум углам и одной стороне определять значения всех параметров рассматриваемой фигуры. В прошлом люди заметили эту закономерность и стали активно ею пользоваться при строительстве зданий, навигации, в астрономии и даже в искусстве.

    Начальный этап

    Первоначально люди рассуждали о взаимоотношении углов и сторон исключительно на примере прямоугольных треугольников. Затем были открыты особые формулы, позволившие расширить границы употребления в повседневной жизни данного раздела математики.

    Изучение тригонометрии в школе сегодня начинается с прямоугольных треугольников, после чего полученные знания используются учениками в физике и решении абстрактных тригонометрических уравнений, работа с которыми начинается в старших классах.

    Сферическая тригонометрия

    Позже, когда наука вышла на следующий уровень развития, формулы с синусом, косинусом, тангенсом, котангенсом стали использоваться в сферической геометрии, где действуют иные правила, а сумма углов в треугольнике всегда больше 180 градусов. Данный раздел не изучается в школе, однако знать о его существовании необходимо как минимум потому, что земная поверхность, да и поверхность любой другой планеты, является выпуклой, а значит, любая разметка поверхности будет в трёхмерном пространстве «дугообразной».

    Возьмите глобус и нитку. Приложите нитку к двум любым точкам на глобусе, чтобы она оказалась натянутой. Обратите внимание — она обрела форму дуги. С такими формами и имеет дело сферическая геометрия, применяющаяся в геодезии, астрономии и других теоретических и прикладных областях.

    Прямоугольный треугольник

    Немного узнав про способы применения тригонометрии, вернемся к базовой тригонометрии, чтобы в дальнейшем разобраться, что такое синус, косинус, тангенс, какие расчёты можно с их помощью выполнять и какие формулы при этом использовать.

    Первым делом необходимо уяснить понятия, относящиеся к прямоугольному треугольнику. Во-первых, гипотенуза — это сторона, лежащая напротив угла в 90 градусов. Она является самой длинной. Мы помним, что по теореме Пифагора её численное значение равно корню из суммы квадратов двух других сторон.

    Например, если две стороны равны 3 и 4 сантиметрам соответственно, длина гипотенузы составит 5 сантиметров. Кстати, об этом знали ещё древние египтяне около четырех с половиной тысяч лет назад.

    Две оставшиеся стороны, которые образуют прямой угол, носят название катетов. Кроме того, надо помнить, что сумма углов в треугольнике в прямоугольной системе координат равняется 180 градусам.

    Определение

    Наконец, твердо понимая геометрическую базу, можно обратиться к определению синуса, косинуса и тангенса угла.

    Синусом угла называется отношение противолежащего катета (т. е. стороны, располагающейся напротив нужного угла) к гипотенузе. Косинусом угла называется отношение прилежащего катета к гипотенузе.

    Запомните, что ни синус, ни косинус не может быть больше единицы! Почему? Потому что гипотенуза — это по умолчанию самая длинная Каким бы длинным ни был катет, он будет короче гипотенузы, а значит, их отношение всегда будет меньше единицы. Таким образом, если у вас в ответе к задаче получился синус или косинус со значением, большим, чем 1, ищите ошибку в расчётах или рассуждениях. Этот ответ однозначно неверен.

    Наконец, тангенсом угла называется отношение противолежащей стороны к прилежащей. Тот же самый результат даст деление синуса на косинус. Посмотрите: в соответствии с формулой мы делим длину стороны на гипотенузу, после чего делим на длину второй стороны и умножаем на гипотенузу. Таким образом, мы получаем то же самое соотношение, что и в определении тангенса.

    Котангенс, соответственно, представляет собой отношение прилежащей к углу стороны к противолежащей. Тот же результат мы получим, разделив единицу на тангенс.

    Итак, мы рассмотрели определения, что такое синус, косинус, тангенс и котангенс, и можем заняться формулами.

    Простейшие формулы

    В тригонометрии не обойтись без формул — как найти синус, косинус, тангенс, котангенс без них? А ведь именно это требуется при решении задач.

    Первая формула, которую необходимо знать, начиная изучать тригонометрию, говорит о том, что сумма квадратов синуса и косинуса угла равна единице. Данная формула является прямым следствием теоремы Пифагора, однако позволяет сэкономить время, если требуется узнать величину угла, а не стороны.

    Многие учащиеся не могут запомнить вторую формулу, также очень популярную при решении школьных задач: сумма единицы и квадрата тангенса угла равна единице, деленной на квадрат косинуса угла. Присмотритесь: ведь это то же самое утверждение, что и в первой формуле, только обе стороны тождества были поделены на квадрат косинуса. Выходит, простая математическая операция делает тригонометрическую формулу совершенно неузнаваемой. Помните: зная, что такое синус, косинус, тангенс и котангенс, правила преобразования и несколько базовых формул вы в любой момент сможете сами вывести требуемые более сложные формулы на листе бумаги.

    Формулы двойного угла и сложения аргументов

    Ещё две формулы, которые требуется выучить, связаны со значениями синуса и косинуса при сумме и разности углов. Они представлены на рисунке ниже. Обратите внимание, что в первом случае оба раза перемножается синус и косинус, а во втором складывается попарное произведение синуса и косинуса.

    Также существуют формулы, связанные с аргументами в виде двойного угла. Они полностью выводятся из предыдущих — в качестве тренировки попробуйте получить их самостоятельно, приняв угол альфа равным углу бета.

    Наконец, обратите внимание, что формулы двойного угла можно преобразовать так, чтобы понизить степень синуса, косинуса, тангенса альфа.

    Теоремы

    Двумя основными теоремами в базовой тригонометрии являются теорема синусов и теорема косинусов. С помощью этих теорем вы легко сможете понять, как найти синус, косинус и тангенс, а значит, и площадь фигуры, и величину каждой стороны и т. д.

    Теорема синусов утверждает, что в результате деления длины каждой из сторон треугольника на величину противолежащего угла мы получим одинаковое число. Более того, это число будет равно двум радиусам описанной окружности, т. е. окружности, содержащей все точки данного треугольника.

    Теорема косинусов обобщает теорему Пифагора, проецируя её на любые треугольники. Оказывается, из суммы квадратов двух сторон вычесть их произведение, умноженное на двойной косинус смежного им угла — полученное значение окажется равно квадрату третьей стороны. Таким образом, теорема Пифагора оказывается частным случаем теоремы косинусов.

    Ошибки по невнимательности

    Даже зная, что такое синус, косинус и тангенс, легко совершить ошибку из-за рассеянности внимания или ошибки в простейших расчётах. Чтобы избежать таких ошибок, ознакомимся с наиболее популярными из них.

    Во-первых, не следует преобразовывать обыкновенные дроби в десятичные до получения окончательного результата — можно и ответ оставить в виде обыкновенной дроби, если в условии не оговорено обратное. Такое преобразование нельзя назвать ошибкой, однако следует помнить, что на каждом этапе задачи могут появиться новые корни, которые по задумке автора должны сократиться. В этом случае вы напрасно потратите время на излишние математические операции. Особенно это актуально для таких значений, как корень из трёх или из двух, ведь они встречаются в задачах на каждом шагу. То же касается округлений «некрасивых» чисел.

    Далее, обратите внимание, что к любому треугольнику применима теорема косинусов, но не теорема Пифагора! Если вы по ошибке забудете вычесть удвоенное произведение сторон, умноженное на косинус угла между ними, вы не только получите совершенно неверный результат, но и продемонстрируете полное непонимание предмета. Это хуже, чем ошибка по невнимательности.

    В-третьих, не путайте значения для углов в 30 и 60 градусов для синусов, косинусов, тангенсов, котангенсов. Запомните эти значения, ведь синус 30 градусов равен косинусу 60, и наоборот. Их легко перепутать, вследствие чего вы неизбежно получите ошибочный результат.

    Применение

    Многие ученики не спешат приступать к изучению тригонометрии, поскольку не понимают её прикладного смысла. Что такое синус, косинус, тангенс для инженера или астронома? Это понятия, благодаря которым можно вычислить расстояние до далёких звёзд, предсказать падение метеорита, отправить исследовательский зонд на другую планету. Без них нельзя построить здание, спроектировать автомобиль, рассчитать нагрузку на поверхность или траекторию движения предмета. И это только самые очевидные примеры! Ведь тригонометрия в том или ином виде используется повсюду, начиная от музыки и заканчивая медициной.

    В заключение

    Итак, вы синус, косинус, тангенс. Вы можете использовать их в расчётах и успешно решать школьные задачи.

    Вся суть тригонометрии сводится к тому, что по известным параметрам треугольника нужно вычислить неизвестные. Всего этих параметров шесть: длины трёх сторон и величины трёх углов. Всё различие в задачах заключается в том, что даются неодинаковые входные данные.

    Как найти синус, косинус, тангенс исходя из известных длин катетов или гипотенузы, вы теперь знаете. Поскольку эти термины обозначают не что иное, как отношение, а отношение — это дробь, главной целью тригонометрической задачи становится нахождение корней обычного уравнения либо же системы уравнений. И здесь вам поможет обычная школьная математика.

    Формулы суммы и разности синусов и косинусов для двух углов α и β позволяют перейти от суммы указанных углов к произведению углов α + β 2 и α — β 2 . Сразу отметим, что не стоит путать формулы суммы и разности синусов и косинусов с формулами синусов и косинусов суммы и разности. Ниже мы перечислим эти формулы, приведем их вывод и покажем примеры применения для конкретных задач.

    Yandex.RTB R-A-339285-1

    Формулы суммы и разности синусов и косинусов

    Запишем, как выглядят формулы суммы и разности для синусов и для косинусов

    Формулы суммы и разности для синусов

    sin α + sin β = 2 sin α + β 2 cos α — β 2 sin α — sin β = 2 sin α — β 2 cos α + β 2

    Формулы суммы и разности для косинусов

    cos α + cos β = 2 cos α + β 2 cos α — β 2 cos α — cos β = — 2 sin α + β 2 cos α — β 2 , cos α — cos β = 2 sin α + β 2 · β — α 2

    Данные формулы справедливы для любых углов α и β . Углы α + β 2 и α — β 2 называются соответственно полусуммой и полуразностью углов альфа и бета. Дадим формулировку для каждой формулы.

    Определения формул сумм и разности синусов и косинусов

    Сумма синусов двух углов равна удвоенному произведению синуса полусуммы этих углов на косинус полуразности.

    Разность синусов двух углов равна удвоенному произведению синуса полуразности этих углов на косинус полусуммы.

    Сумма косинусов двух углов равна удвоенному произведению косинуса полусуммы и косинуса полуразности этих углов.

    Разность косинусов двух углов равна удвоенному произведению синуса полусуммы на косинус полуразности этих углов, взятому с отрицательным знаком.

    Вывод формул суммы и разности синусов и косинусов

    Для вывода формул суммы и разности синуса и косинуса двух углов используются формулы сложения. Приведем их ниже

    sin (α + β) = sin α · cos β + cos α · sin β sin (α — β) = sin α · cos β — cos α · sin β cos (α + β) = cos α · cos β — sin α · sin β cos (α — β) = cos α · cos β + sin α · sin β

    Также представим сами углы в виде суммы полусумм и полуразностей.

    α = α + β 2 + α — β 2 = α 2 + β 2 + α 2 — β 2 β = α + β 2 — α — β 2 = α 2 + β 2 — α 2 + β 2

    Переходим непосредственно к выводу формул суммы и разности для sin и cos.

    Вывод формулы суммы синусов

    В сумме sin α + sin β заменим α и β на выражения для этих углов, приведенные выше. Получим

    sin α + sin β = sin α + β 2 + α — β 2 + sin α + β 2 — α — β 2

    Теперь к первому выражению применяем формулу сложения, а ко второму — формулу синуса разностей углов (см. формулы выше)

    sin α + β 2 + α — β 2 = sin α + β 2 cos α — β 2 + cos α + β 2 sin α — β 2 sin α + β 2 — α — β 2 = sin α + β 2 cos α — β 2 — cos α + β 2 sin α — β 2 sin α + β 2 + α — β 2 + sin α + β 2 — α — β 2 = sin α + β 2 cos α — β 2 + cos α + β 2 sin α — β 2 + sin α + β 2 cos α — β 2 — cos α + β 2 sin α — β 2 Раскроем скобки, приведем подобные слагаемые и получим искомую формулу

    sin α + β 2 cos α — β 2 + cos α + β 2 sin α — β 2 + sin α + β 2 cos α — β 2 — cos α + β 2 sin α — β 2 = = 2 sin α + β 2 cos α — β 2

    Действия по выводу остальных формул аналогичны.

    Вывод формулы разности синусов

    sin α — sin β = sin α + β 2 + α — β 2 — sin α + β 2 — α — β 2 sin α + β 2 + α — β 2 — sin α + β 2 — α — β 2 = sin α + β 2 cos α — β 2 + cos α + β 2 sin α — β 2 — sin α + β 2 cos α — β 2 — cos α + β 2 sin α — β 2 = = 2 sin α — β 2 cos α + β 2

    Вывод формулы суммы косинусов

    cos α + cos β = cos α + β 2 + α — β 2 + cos α + β 2 — α — β 2 cos α + β 2 + α — β 2 + cos α + β 2 — α — β 2 = cos α + β 2 cos α — β 2 — sin α + β 2 sin α — β 2 + cos α + β 2 cos α — β 2 + sin α + β 2 sin α — β 2 = = 2 cos α + β 2 cos α — β 2

    Вывод формулы разности косинусов

    cos α — cos β = cos α + β 2 + α — β 2 — cos α + β 2 — α — β 2 cos α + β 2 + α — β 2 — cos α + β 2 — α — β 2 = cos α + β 2 cos α — β 2 — sin α + β 2 sin α — β 2 — cos α + β 2 cos α — β 2 + sin α + β 2 sin α — β 2 = = — 2 sin α + β 2 sin α — β 2

    Примеры решения практических задач

    Для начала, сделаем проверку одной из формул, подставив в нее конкретные значения углов. Пусть α = π 2 , β = π 6 . Вычислим значение суммы синусов этих углов. Сначала воспользуемся таблицей основных значений тригонометрических функций, а затем применим формулу для суммы синусов.

    Пример 1. Проверка формулы суммы синусов двух углов

    α = π 2 , β = π 6 sin π 2 + sin π 6 = 1 + 1 2 = 3 2 sin π 2 + sin π 6 = 2 sin π 2 + π 6 2 cos π 2 — π 6 2 = 2 sin π 3 cos π 6 = 2 · 3 2 · 3 2 = 3 2

    Рассмотрим теперь случай, когда значения углов отличаются от основных значений, представленных в таблице. Пусть α = 165 ° , β = 75 ° . Вычислим значение разности синусов этих углов.

    Пример 2. Применение формулы разности синусов

    α = 165 ° , β = 75 ° sin α — sin β = sin 165 ° — sin 75 ° sin 165 — sin 75 = 2 · sin 165 ° — sin 75 ° 2 cos 165 ° + sin 75 ° 2 = = 2 · sin 45 ° · cos 120 ° = 2 · 2 2 · — 1 2 = 2 2

    С помощью формул суммы и разности синусов и косинусов можно перейти от суммы или разности к произведению тригонометрических функций. Часто эти формулы называют формулами перехода от суммы к произведению. Формулы суммы и разности синусов и косинусов широко используются при решении тригонометрических уравнений и при преобразовании тригонометрических выражений.

    Если вы заметили ошибку в тексте, пожалуйста, выделите её и нажмите Ctrl+Enter

    Использование предлога in в английском языке

    Употребление и произношение in

    Сводка тригонометрических формул

    Сводка тригонометрических формул

    Эти формулы связывают длины и площади определенных кругов или треугольников. На следующей странице вы найдете личности. Тождества не относятся к конкретным геометрическим фигурам, но справедливы для всех углов.

    Формулы для дуг и секторов окружностей

    Вы можете легко найти длину дуги и площадь сектора для угла θ в круге радиусом r .

    Длина дуги. Длина дуги равна радиусу r , умноженному на угол θ , где угол измеряется в радианах. Чтобы перевести градусы в радианы, умножьте количество градусов на π /180.
    Площадь сектора. Площадь сектора равна половине квадрата радиуса, умноженного на угол, где, опять же, угол измеряется в радианах.
    Формулы для прямоугольных треугольников

    Наиболее важными формулами тригонометрии являются формулы прямоугольного треугольника. Если θ — один из острых углов в треугольнике, то синус теты — это отношение противолежащего катета к гипотенузе, косинус — отношение прилежащего катета к гипотенузе, а тангенс — это отношение противоположная сторона соседней стороне.

    Эти три формулы вместе известны мнемоникой SohCahToa. Кроме того, есть очень важная формула Пифагора, которая гласит, что квадрат гипотенузы равен сумме квадратов двух других сторон.

    Зная, что два острых угла дополняют друг друга, то есть дают в сумме 90°, можно решить любой прямоугольный треугольник:

    • Если вы знаете две из трех сторон, вы можете найти третью сторону и оба острых угла.
    • Если вы знаете один острый угол и одну из трех сторон, вы можете найти другой острый угол и две другие стороны.
    Формулы для косоугольных треугольников

    Эти формулы работают для любого треугольника, острого, тупоугольного или прямоугольного. Мы будем использовать стандартные обозначения, в которых три вершины треугольника обозначены прописными буквами A , B и C , а три противоположные им стороны соответственно обозначены строчными буквами a , б и в .

    Есть две важные формулы для косых треугольников. Они называются законом косинусов и законом синусов.

    Закон косинусов обобщает формулу Пифагора на все треугольники. Он говорит, что c 2 , квадрат одной стороны треугольника, равен a 2  +  b 2 , сумма квадратов двух других сторон минус 2. ab  cos&nbsp C , удвоенное произведение их на косинус противоположного угла. Когда угол C правильный, он становится формулой Пифагора.

    Закон синусов гласит, что отношение синуса одного угла к противоположной стороне одинаково для всех трех углов.

    С помощью этих двух формул вы можете решить любой треугольник:

    • Если вам известны два угла и сторона, вы можете найти третий угол и две другие стороны.
    • Если известны две стороны и угол между ними, можно найти третью сторону и оба других угла.
    • Если вы знаете две стороны и угол, противолежащий одной из них, то существует два варианта угла, противоположного другому (один острый и один тупой), и для обоих вариантов вы можете определить оставшийся угол и оставшуюся сторону.
    Формулы площади треугольников

    Существует три различных полезных формулы площади треугольника, и какую из них использовать, зависит от имеющейся у вас информации.

    Половина основания, умноженная на высоту. Это обычный способ использования, так как он самый простой, и у вас обычно есть эта информация. Выберите любую сторону для вызова базы b . Тогда, если h — это расстояние от противоположной вершины до b , то площадь равна половине bh .
    Формула Герона. Это полезно, когда вы знаете три стороны a , b и c треугольника, и все, что вы хотите знать, это площадь. Пусть s составляют половину их суммы, называемой полупериметром . Тогда площадь равна квадратному корню из произведения s , s  —  a , s  —  b и s  —  c .
    Формула «бок-угол-бок». Используйте это, когда вы знаете две стороны, a и b , и прилежащий угол, C . Площадь равна половине произведения двух сторон на синус прилежащего угла.

    Формулы Sin Cos – вывод, примеры

    Основные тригонометрические функции – это формулы sin и cos, которые относятся к углам и отношениям сторон прямоугольного треугольника. Синус угла есть отношение противолежащего катета к гипотенузе, а косинус угла есть отношение прилежащего катета к гипотенузе. Они образуют фундаментальные тождества, которые определены для острых углов. Распространение этих отношений на любой угол в радианах называется тригонометрической функцией. Sin положителен в первом и втором квадранте, а cos положителен в первом и четвертом квадранте. Диапазон функций синуса и косинуса составляет [-1,1] в области действительных чисел.

    Что такое формулы Sin Cos?

    Если (x, y) — точка на единичной окружности, и если луч из начала координат (0, 0) в (x, y) образует угол θ с положительной осью, то x и y удовлетворяют пифагорову Теорема x 2 + y 2 = 1, где x и y образуют длины катетов прямоугольного треугольника. Таким образом, основная формула sin cos становится следующей: cos 2 θ + sin 2 θ = 1.

    Существует множество тождеств, связанных с синусом и косинусом, которые применяются в тригонометрических функциях. Все тригонометрические выражения проще вычислить с помощью этих тригонометрических формул. Давайте обсудим их подробно.

    Формулы Sin Cos

    Для любого острого угла θ функции отрицательных углов:

    • sin(-θ) = – sinθ
    • cos (-θ) = cosθ

    Тождества, выражающие триггерные функции через их дополнения:

    • cosθ = sin(90° — θ)
    • sinθ = cos(90 ° — θ)

    Сумма и разность формул Sin Cos

    Угол, составленный из суммы или разности двух или более углов, называется составным углом. Обозначим составные углы как α и β. Существуют формулы Sin Cos относительно сложных углов для расширения или упрощения тригонометрических выражений. Давайте исследуем их.

    • sin (α + β) = sin α cos β + cos α sin β
    • sin (α – β) = sin α cos β – cos α sin β
    • cos (α + β) = cos α cos β – sin α sin β
    • cos (α – β) = cos α cos β + sin α sin β

    Преобразование формул Sin и Cos

    Есть несколько тождеств, которые мы выбираем с одной стороны для работы и делаем замены до тех пор, пока сторона не преобразуется в другую. Чтобы проверить тождество, мы перепишем любую часть уравнения и преобразуем ее в другую часть. Из вышеупомянутых тождеств суммы и разности мы получаем формулы произведения на сумму и суммы на произведение.

    Формулы произведения на сумму применяются, когда задано произведение косинусов. Мы выражаем произведение в виде суммы или разности, пишем формулу, подставляем заданные углы и, наконец, упрощаем.

    • 2 sin α cos β = sin (α + β) + sin (α – β)
    • 2 cos α sin β = sin (α + β) – sin (α – β)
    • 2 cos α cos β = cos (α + β) + cos (α – β)
    • 2 sin α sin β = cos (α – β) – cos (α + β)

    Формулы преобразования суммы в произведение позволяют выражать суммы синуса или косинуса в виде произведений. Эти формулы приведены ниже:

    • sin α + sin β = 2 sin((α+β)/2) cos((α−β)/2)
    • sin α – sin β = 2 cos((α+β)/2) sin((α−β)/2)
    • cos α + cos β= 2 cos((α+β)/2) cos((α−β)/2)
    • cos α – cos β = -2 sin((α+β)/2) sin((α-β)/2)

    Вывод произведения по формуле суммирования

    Здесь мы выражаем произведения косинуса и синуса в виде суммы. Мы можем вывести формулу произведения на сумму из тождеств суммы и разности для косинуса. Если мы сложим два уравнения, то получим:

       cosα cosβ + sinα sinβ = cos(α − β)
    + cosα cosβ − sinα sinβ = cos(α + β)
    ———————————————
    2cosα cosβ = cos(α−β) + cos(α + β)
    ——————————-

    Затем разделите на 2 и выделить произведение косинусов:  cosα cosβ = (1/2)[cos(α−β) + cos(α+β)]

    Точно так же мы можем вывести другие формулы, представив произведения в виде суммы/разности.

      Вывод суммы из формулы произведения 

      Есть несколько задач, которые требуют обращения произведения к сумме. Давайте посмотрим на вывод этих формул суммы в произведение. Для этого сделаем несколько замен, например (u+v)/2 = α, (u-v)/2 = β

      Тогда α + β = [(u+v)/2] + [(u-v )/2] = u

      α — β = [(u + v)/2] — [(u — v)/2] = v

      Выведем формулу произведения суммы. Мы заменяем α и β в формуле произведения на сумму.

      Рассмотрим (sinα cosβ) = (1/2)[sin(α + β) + sin(α — β)]

      Заменив (α + β) и αβ, мы получим

      sin((u+v )/2) cos ((u-v)/2) = 1/2[sinu + sin v]

      2sin((u+v)/2)) cos ((u-v)/2) = sinu + sin v

      Аналогичным образом мы можем вывести другие тождества суммы и произведения.

      Sin Cos Формулы кратных углов

      У нас есть формулы двойных и тройных углов, а также формулы половинного угла:

      • sin 2θ = 2 sinθ cosθ
      • sin 3θ = 3 sinθ — 4 sin 3 θ
      • cos 2θ = cos 2  θ — sin 2 θ
      • cos 2θ = 2cos 2 θ — 1
      • cos 2θ = 1- 2sin 2 θ
      • cos 3θ = 4 cos 3 θ — 3cosθ
      • sin (θ/2) = ± √((1- cosθ)/2)
      • cos (θ/2) = ± √((1+ cosθ)/2)
      • sin θ = 2tan  ( θ/2) /(1 + tan (θ/2))
      • cos θ = (1-тангенс 2 (θ/2))/(1 + тангенс 2 (θ/2))

       

      Разбивайте сложные концепции с помощью простых визуальных средств.

      Математика больше не будет сложным предметом, особенно когда вы понимаете концепции с помощью визуализаций с помощью Cuemath.

      Забронировать бесплатный пробный урок

       

      Примеры использования формул sin-cos

      Пример 1: Когда sin X = 1/2 и cos Y = 3/4, найдите cos(X+Y)

      Решение: Мы знаем cos(X + Y) ) = cos X cos Y – sin X sin Y

      Учитывая sin X = 1/2

      Мы знаем, что cos X = √(1 — sin 2 X) = √(1 — (1/4)) = √3/2

      Таким образом, cos X = √3/2

      Учитывая cos Y = 3/4

      Мы знаем, что sin Y = √(1 — cos 2 Y) = √(1 — (9/16)) = √7/4

      Таким образом, sin Y = √7 /4

      cos X = √3/2, а sinY = √7/4

      Применяя формулу суммы cos, имеем cos(X+Y) = ( √3/2) × (3/4) –  1/2 × (√7/4)

      = (3√3 — √7)/8

      Ответ: cos(X+Y) = (3√ 3 — √7)/8

      Пример 2: Если sin θ = 3/5, найдите sin2θ.

      Решение: Мы знаем, что sin2θ = 2 sin θ cos θ

      Нам нужно определить cos θ.

      Воспользуемся формулой sin cos cos 2 θ + sin 2 θ = 1.

      Переписать, мы получаем COS 2 θ = 1 -SIN 2 θ

      = 1- (9/25)

      COS 2 θ = 16/25

      θ = 4/5

      sin2θ = 2 sin θ cos θ

      = 2 × (3/5) × (4/5) = 24/25

      Ответ: sin2θ = 24/25

      39193 Пример 3 : Докажите (cos 4a — cos 2a)/ (sin 4a + sin 2a) = -tan a.

      Решение: Используя формулу sin cos, давайте перепишем левый градус и преобразуем его в правый

      \(=\dfrac{-2\sin(\dfrac{4a+2a}{2})\sin(\dfrac{4a-2a}{2})}{2\sin(\dfrac{4a+2a {2})\cos(\dfrac{4a-2a}{2})}\)

      \(=\dfrac{-2\sin(3a) sina}{2\sin(3a) cosa}\)

      = — sina / cosa

      = −tan a

      Таким образом доказано.

      Ответ: (cos 4a — cos 2a)/ (sin 4a + sin 2a) = -tan a.

      Часто задаваемые вопросы о формулах Sin Cos

      Что такое формулы Sin Cos?

      В прямоугольном треугольнике сторона, противоположная прямому углу, является гипотенузой, а два катета являются прилежащей и противолежащей сторонами. Тогда тригонометрические отношения задаются как cosθ = смежный / гипотенуза и sinθ = противолежащий / гипотенуза.

      Чему равен Sinθ/Cosθ?

      Отношение синуса к косинусу равно тангенсу того же угла, tanθ = sinθ/cosθ.

      Как отличить Cos от Sin?

      В любом прямоугольном треугольнике синус есть противолежащая сторона/гипотенуза. Таким образом, зная эти две стороны, соседняя сторона находится и применяется в формуле косинуса, которая является смежной стороной / гипотенузой.

      Чему равен Cos?

      Косинус угла равен синусу дополнительного угла. cos θ = sin(90°-θ).

      Законы синусов и формул косинусов

      Тригонометрия — это раздел математики, связанный с концепцией оценки углов. Чаще всего эти углы измеряются в прямоугольном треугольнике. Тригонометрические функции получаются с помощью тригонометрических операций, которые выполняются со сторонами и углами прямоугольного треугольника соответственно.

      Данная статья посвящена подтеме тригонометрии. Статья объясняет закон формулы синусов и закон формулы косинуса, а содержание также включает объяснение тригонометрических отношений и их тригонометрических значений по отношению к различным углам.

      Что такое тригонометрические отношения?

      Тригонометрические отношения – это значения тригонометрических функций, представленные в числовой форме и полученные в виде отношений сторон и углов данного прямоугольного треугольника. В прямоугольном треугольнике три стороны, т. гипотенуза, перпендикуляр и основание, от которых зависят отношения. Шесть основных функций тригонометрии, которые дают разные числовые значения под разными углами, — это синус, косинус, тангенс, косеканс, секанс и котангенс.

      The trigonometric values ​​of these functions under different angles are given below:

      Functions 30° 45° 60° 90°
      sinθ 0 1/2 1/√2 √3/2 1
      cosθ 1 √3/2 1/√2 1/2 0
      tanθ 0 1/√3 1 √3
      cotθ √3 1 1/√3 0
      secθ 1 2√3 √2 2
      cosecθ 2 √2 2√3 1

      Law of Sines Formula

      Формула закона синусов или правило синусов понимается как отношение сторон и эквивалентных углов данного треугольника. Формула закона синусов обычно используется для оценки неизвестных сторон или углов косоугольного треугольника.

      Математическая формула закона синусов задается как

      , где a, b и c — стороны треугольника.

      В тригонометрической операции, где используется правило синусов, мы обычно рассматриваем как минимум два угла треугольника.

      Вывод формулы закона синусов

      Прямоугольный треугольник считается доказательством формулы правила синусов.

      Итак, предположим, что треугольник ABC с соответствующими сторонами

      AB = c

      BC = a

      и, AC = b

      Затем проведите перпендикуляр CD от основания треугольника, т.е. AB. Следовательно, высота данного треугольника будет CD = h.

      Теперь проведенный перпендикуляр разделит треугольник на два прямоугольных треугольника, которые будут CDB и CDA.

      Здесь для оценки a/b=sinA/SinB

      В CDA

      =>sinA=h/b

      И, В CDB

      =>sinB=h/a

      Тогда,

      =>sinA/sinB=(h/b)/(h/a)

      Следовательно,

      => SinA/sinB=a/b доказано.

      Точно так же другие функции как sinB/sinC = b/c также могут быть доказаны с применением того же метода.

      Некоторые другие формулы правила синусов

    Формула закона косинуса

    Закон косинуса или правило косинуса – это выражение, связывающее длины сторон и косинусы углов данного треугольника. Закон косинуса гласит, что «квадрат любой стороны треугольника равен разности между суммой квадратов других сторон и удвоенным произведением других сторон и косинуса угла между ними».

    Математически закон косинуса выражается как

    a 2 = b 2 + c 2 – 2bc. cosA

    b 2 = c 2 + a 2 – 2ca. cosB

    c 2 = a 2 + b 2 – 2ab. cosC

    где,

    a,b и c — длины сторон

    And, A, B и C — углы.

    Вывод закона косинуса

    Предположим, что треугольник ABC имеет соответствующие стороны

    AB = c

    BC = a

    и AC = b

    . Затем из основания треугольника, т. е. AC, проведите перпендикуляр OB. Следовательно, высота данного треугольника будет OB=h.

    Теперь проведенный перпендикуляр разделит треугольник на два прямоугольных треугольника, которые будут ABO и BOC.

    Сейчас, В ABO

    sinA=BO/AB=h/c…………..(i)

    и,

    cosA=AO/AB=d/c ………….(ii)

    Из уравнений (i) и (ii) получаем значение h и d,

    h=c(sinA)

    d= c(cosA)

    Используя теорему Пифагора в BOC

    =>a 2 =h 2 +(b-d) 2

    Подставляя значения h и d, мы получаем

    3 =>

    3 2 =c 2 sin 2 A+b 2 +c 2 cos 2 A-2bc. cosA

    =>а 2 2 (sin 2 A+cos 2 A)+b 2 -2bc.cosA

    => a 2 =c 2 +b 2 +b 2 cosA доказал.

    Подобным образом можно доказать и другие выражения

    b=c+a-2ca. cosB и

    c=a+b-2ab.cosC тем же методом.

    Примеры задач

    Задача 1. Найдите недостающую сторону треугольника, две стороны которого равны 12 см и 8 см, а угол между ними равен 60 градусам.

    Решение:

    Дано

    Пусть две стороны b и c равны 12 см и 8 см соответственно, а сторона a недостающая.

    Угол между b и c равен 60°

    Теперь, используя формулу закона косинуса

    a 2 =b 2 +c 2 -2bc. cosA

    =>a 2 =(12) 2 +(8) 2 -2(12)(8)cos60°

    =>a 2 =144+64-19900,5

    => 2 =208-96

    =>a 2 =112

    =>a=10,58 см

    Задача 2. Найдите недостающую сторону треугольника, две стороны которого равны 25 см и 10 см, а угол между ними измерения 30градусов.

    Решение:

    Дано

    Пусть две стороны b и c равны 25 см и 10 см соответственно, а сторона a будет отсутствующей.

    Угол между b и c равен 30°.

    Теперь, используя формулу закона косинуса

    a 2 =b 2 +c 2 -2bc. cosA

    =>a 2 =(25) 2 +(10) 2 -2(25)(10)cos30°

    =>a 2 =600+100-6000

    =>a 2 =725-430

    =>a 2 =295

    =>a=17,17 см

    с углом между ними 45 градусов.

    Решение:

    Дано

    Пусть две стороны b и c равны 21 см и 14 см соответственно, а сторона a — недостающая.

    Угол между b и c равен 45°.

    Теперь, используя формулу закона косинуса

    a 2 =b 2 +c 2 -2bc. cosA

    =>a 2 =(21) 2 +(14) 2 -2(21)(14)cos45°

    =>a 2 =441+197-0788

    =>а 2 =637-415,7

    =>a 2 =221,3

    =>a=14,87см

    Задача 4: Найдите значение стороны a, если два угла равны ∠A=65° и ∠B=40° и b =12см.

    Решение:

    дано

    ♂ = 65 ° и ♂ = 40 °

    и, b = 12см

    Теперь,

    a/sina = b/b/sinb

    => а////// sin65°=12/sin40°

    =>a/0,906=12/0,642

    =>a=16,93см.

    Задача 5: Найдите значение стороны a, если два угла равны ∠A=72° и ∠B=60° и b=6см.

    Решение:

    дано

    ♂ = 72 ° и ♂ = 60 °

    и, b = 6cm

    Теперь,

    a/sina = b/snb

    => а/////////// sin72°=6/sin60°

    =>a/0,951=6/0,86

    =>a=6,41см.

    Задача 6: Найдите значение стороны a, если два угла равны ∠A=48° и ∠B=35° и b=16см.

    Решение:

    Дано

    ∠A=48° и ∠B=35°

    А, b=16см

    Теперь,

    a/sinA=b/sinB

    =>a/sin48°=16/sin35°

    =>a/0,743=16/0,57

    =>a=20,85см.

    исчисление — Теорема синусов и косинусов — формулы суммы и разности

    спросил

    Изменено 1 год, 5 месяцев назад

    Просмотрено 6к раз

    $\begingroup$

    Я где-то читал, что функции синуса и косинуса могут быть полностью описаны этой теоремой:

    1. $\sin(0) = 0, \cos(0) = 1$
    2. $\sin(a-b) = \sin(a)\cos(b) — \sin(b)\cos(a)$
    3. $\cos(a-b) = \cos(a)\cos(b) + \sin(a)\sin(b)$
    4. Существует na $r>0$ такое, что: $$0<\sin(x)

    С помощью этой теоремы мы можем доказать такие вещи, как: 92(x) = 1$ , выполнив $\cos(a-a) = \cos(a)\cos(a) + \sin(a)\sin(a)$

  • $\sin(-x) = -\sin(x)$ , выполнив $\sin(0-x) = \sin(0)\cos(x) — \sin(x)\cos(0 )$

  • $\cos(a+b) = \cos(a)\cos(b) — \sin(a)\sin(b)$

    выполнив

    $\cos(a-(-b)) = \cos(a)\cos(-b) + \sin(a)\sin(-b)$

  • $\sin(a+b) = \sin(a)\cos(b)+\sin(b)\cos(a)$

    выполнив

    $\sin(a-(-b)) = \sin(a)\cos(-b) — \sin(-b)\cos(a)$
  • И другие тригонометрические тождества, которые также вытекают из того, что я уже сделал.

    Проблема в том, что существует множество определений функций синуса и косинуса. Начнем с классического определения:

    Классическое определение

    Функция синуса определяется как отношение между противоположной стороной угла и гипотенузой этого прямоугольного треугольника.

    Функция косинуса определяется как отношение между смежной стороной угла и гипотенузой этого прямоугольного треугольника.

    Касательная функция определяется как отношение между функцией синуса и функцией косинуса (с $\cos (x) \neq 0)$

    Другие тригонометрические тождества могут быть доказаны геометрически для угол меньше или равен $\frac{\pi}{2}rad$ , потому что это прямоугольный треугольник. Таким образом, мы не можем доказать $\sin(a-b)$ геометрически, а затем доказать $\sin(a+b)$ аналитически, как это сделал я, потому что мы предположили отрицательное значение $b$, которое не определено геометрически в прямоугольном треугольнике. . 92(x) = 1$ можно доказать с помощью простой теоремы Пифагора для треугольника с гипотенузой 1.

    Формулы суммы и разности синусов и косинусов можно доказать геометрически, как на этих изображениях, которые я нашел в этом ответе:

    Определение единичной окружности

    Представьте себе окружность с центром в начале декартовой плоскости, тогда:

    Функция синуса для $x \in R, x>0$ может быть определена как $y$ положение точки окружности, где угол заканчивается, если мы путешествие против часовой стрелки внутри линии круга.

    Косинус для функции $x \in R, x>0$ может быть определен как $x$ положение точки окружности, где угол заканчивается, если мы перемещаемся против часовой стрелки внутри окружности линия.

    Мы можем дать такое же определение для отрицательных углов, так что для $x \in R, x<0$ то же верно, но мы знаем, что перемещается по по часовой стрелке.

    Тангенс 9Функция 1107 определяется как отношение между функцией синуса и функцией косинуса (с $\cos (x) \neq 0)$

    Затем мы можем определить эти функции для всех действительных чисел, так как когда мы проходим $2\pi$, мы возвращаемся в исходную точку. Итак, мы определили синус и косинус как периодических функций.

    Открытый вопрос : Как я могу доказать, с периодическим определением единичного круга 9{2n-1}$$

    Здесь, в том же вопросе, есть аналитическое доказательство тригонометрических тождеств для этих сумм.

    Какое определение лучше всего подходит для исчисления?

    Ну, в исчислении мы используем МНОГО тригонометрических функций: в целочисленных заменах, в рядах, рядах Тейлора (таких как те, которые я показал сейчас), производных, признаках сходимости (таких как эйлерова волна в базельской задаче) и других вещах. . .

    Все определения, которые я вижу, являются своего рода круговыми или недостаточно строгими, чтобы я чувствовал себя хорошо, принимая некоторые производные или целочисленные замены, потому что меня всегда волнует область этих вещей. Так что я хочу очень хорошо определить его и иметь возможность использовать все тригонометрические тождества.

    Я видел много геометрических доказательств $\sin(a-b)$, $\sin(a+b)$, $\cos(a-b)$, $\cos(a+b)$ с использованием прямоугольного треугольника и затем внезапно человек начинает использовать эту формулу для всех действительных чисел. Мне нужно полное определение тригонометрических функций, которое работает периодически и для всех вещественных чисел. Определение ряда Тейлора кажется хорошим, но они генерируются с использованием тригонометрических тождеств, которые еще не доказаны (при условии этого определения).

    ps: я знаю, что использовал некоторые примитивные слова в некоторых определениях, например ' проезд ' так что я их в упор , но надеюсь вы понимаете. И извините за длинный пост, но мне нужно было это сделать, потому что я никогда не видел полного определения ни в одной книге. Спасибо.

    • исчисление
    • алгебра-предварительное исчисление
    • геометрия

    $\endgroup$

    6

    $\begingroup$

    Стандартное определение триггерных функций - это единичный круг. Классическое определение в терминах прямоугольных треугольников более ограничено по объему. По этой причине в исчислении используется более надежное определение в терминах единичного круга. Из этого «стандартного» определения можно доказать любое тождество тригонометрических функций.

    В своем посте выше вы дали определение в терминах единичного круга, но оставили то, что я считаю важным моментом. При определении в терминах единичного круга триггерные функции имеют «действительное значение». Здесь мы имеем в виду, что они определяются в терминах расстояния, а не какой-то сконструированной идеи, называемой углом. Итак, позвольте мне начать с определения в терминах единичного круга, затем мы перейдем к родственному доказательству того, что дано выше для синуса разности. Отсюда мы можем установить остальные три тождества. 92 = 1 \end{уравнение} $$ Начиная с точки пересечения x (1,0), мы проходим расстояние t по окружности против часовой стрелки (движению по часовой стрелке будет присвоен отрицательный знак). Как только мы проходим расстояние t по окружности, мы заканчиваем ее в точке P(x,y) на окружности. Затем мы определяем $$ \begin{eqnarray} \cos t = x \\ \sin t = y \end{eqnarray} $$

    Тогда оказывается, что мы можем вернуться к углам, заметив, что на единичной окружности радианная мера центрального угла, опирающегося на дугу длины t, равна t. Но нам никогда не нужно возвращаться к углам. Наше определение стоит особняком. Непосредственным следствием нашего определения является знакомое пифагорейское тождество синуса и косинуса. 92 t = 1 \end{equation} $$
    (Это получается путем подстановки синуса и косинуса в уравнение единичной окружности для x и y.)

    Прежде чем мы перейдем к тождеству, о котором вы спрашивали, сначала отметим, что синус и функции косинуса являются периодическими. IE. $$ \cos(a + 2k \pi) = \cos a $$. Я считаю, что это довольно ясно, потому что любое путешествие по кругу вернет вас туда, откуда вы начали. Совершите это путешествие k раз, и вы все равно вернетесь к тому, с чего начали. По этой причине, как упоминалось выше, мы можем считать, что оба угла равны $$ [0, 2 \pi)$$. Также обратите внимание, что $$\cos (-t) = \cos t $$ для всех действительных чисел t.

    Последнее, на что следует обратить внимание, это то, что вращение является жестким преобразованием. Другими словами, когда мы вращаем плоскость вокруг точки, расстояние между двумя точками до поворота остается таким же после поворота.

    Теперь вернемся к задаче. Покажите, что
    $$\cos(a-b) = \cos a \cos b + \sin a \sin b $$
    Без ограничения общности предположим $$0

    Теперь рассмотрим точки (см. диаграммы) P($\cos a, \sin b)$ и Q($\cos b, \sin b)$ на единичной окружности. Мы можем использовать формулу расстояния, чтобы найти длину отрезка PQ. Затем мы поворачиваем всю плоскость по часовой стрелке на расстояние b по окружности окружности (или на угол с мерой b в радианах). Это поместит точку Q в Q`(1,0), а P теперь будет расстоянием b-a по окружности от (1,0), поэтому она будет в P`($\cos(a-b), \sin(a-b) )$. Теперь перейдем к использованию формулы расстояния. Поскольку расстояние сохраняется за счет поворота, отрезок PQ и отрезок P`Q` будут иметь одинаковую длину. 92(a-b) + 1 - 2 \cos(a-b) \\ - 2 \cos a \cos b - 2 \sin a \sin b = - 2 \cos(a-b) \\ \cos a \cos b \sin a \sin b = \cos(a-b) \end{eqnarray} $$

    Теперь я предположил, что $b

    Вооружившись формулой косинуса разности, теперь мы можем доказать следующее.

    $$ \begin{eqnarray} \cos(a+b) = \cos(a-(-b)) = \cos a \cos(-b) - \sin a \sin(-b) \\ = \cos a \cos b - \sin a \sin b \end{eqnarray} $$

    Последний шаг в равенстве исходит из нашего тождества для $\cos(-t)$ выше и того факта, что $\sin (-t) = -\sin(t) $ как было упомянуто в предыдущем посте.

    Теперь мы можем показать, что $\cos(\frac{\pi}{2} - x) = \sin x$ и так же, как в предыдущей задаче, что $\sin(\frac{\pi}{2 } - х) = потому что х$. Это ваши знакомые кофункциональные личности. Наконец, вооружившись этими уравнениями, мы теперь можем продемонстрировать синус разности, который затем может доказать синус формулы суммы.

    $$ \begin{eqnarray} \sin(a-b) = \cos(\frac{\pi}{2} - (a-b)) \\ = \cos((\frac{\pi}{2} -a ) + b) \\ = \cos(\frac{\pi}{2} -a) \cos b - \sin(\frac{\pi}{2} -a) \sin b \\ = \sin a \cos b - \cos a \sin b \end{eqnarray} $$

    Вот и все. Прошу прощения, если форматирование немного сбилось, все еще привыкаю к ​​системе. Но теперь из этого тождества вы можете доказать почти любое другое базовое тождество, включающее триггерные функции. Все ваши формулы двойного и половинного угла и ваши формулы сумма-произведение и сумма произведения.

    $\endgroup$

    3

    $\begingroup$

    I. \circ,9\circ]$, заметив, что это единственный способ, при котором закон синусов будет верным также и для тупоугольных треугольников. Закон сложения — это теорема Птолемея для некоторого диапазона углов, а затем выбираются расширения, чтобы сделать ее применимой везде.

    В качестве альтернативы можно вывести интегральную формулу площади круглого сектора, а затем использовать эту функцию для определения синуса и косинуса. Это сделано тщательно в главе 15 Спивака исчисления .

    II. Теперь, когда у вас есть все на такой прочной основе, вы можете обновить: выбрать какое-то доказанное свойство синуса и/или косинуса, которое полностью определяет эти функции, принять его как новое определение этих функций и перестроить теорию на этом основании. Конечно, это имеет смысл только в том случае, если вы уже знаете, что интересующие вас функции обладают этими свойствами (потому что вы доказали это из старого определения), поскольку в противном случае вам остается только гадать, откуда мы знаем, что эти вновь определенные функции являются те же, что и те, что мы уже знали, — но внутренне держится своей логикой. Иногда такого рода обновление может сделать теорию более элегантной, иногда позволяет использовать функции в новом контексте, где старое определение бессмысленно, иногда это просто красиво. Вот несколько таких свойств:

    Во-первых, синус — единственная функция $f$, удовлетворяющая

    • $f''=-f$
    • $f(0)=0$
    • $f'(0)=1$

    (См. Теорему Спивака 4. Он доказывает закон сложения не из своего определения, а из этих свойств, причем очень аккуратно. Его определение можно рассматривать как чисто техническое доказательство того, что существует некоторая функция, обладающая этими свойствами, которая также может быть обработана теорией дифференциальных уравнений.)

    Во-вторых, вы можете определить синус и косинус их степенными рядами. Это очень хороший подход, если вы собираетесь использовать их с комплексными числами. 9{-ix}}{2i} $$ поэтому, если вы каким-то образом определили сложную экспоненту, вы можете получить триггерные функции. (Разумеется, определение комплексной экспоненты требует во многом тех же соображений.)

    В-четвертых, для положительного действительного числа $p$ существует ровно одна пара функций $S,C$, удовлетворяющих

    • $C(x-y) =С(х)С(у)+S(х)S(у)$
    • $S(p)=1$
    • $S(x)\ge 0$ для всех $x\in [0,p]$.

    Это основной результат Г. Б. Робисона, "Новый подход к круговым функциям, $\pi$ и $\lim (\sin x)/x$", Math. Маг. 41 (1968), 66–70 (jstor). Взятие $p=\frac\pi2$ дает обычные функции синуса и косинуса. (На самом деле автор предлагает сначала взять за $S$ и $C$ функции, полученные при $p=1$, а затем определить $\pi = 2\lim_{x\to 0} S(x)/x$. ) В статье доказываются все обычные базовые тригонометрические тождества, за исключением, конечно, тождества для $\cos(x-y)$, так как это определение!

    Робисон ссылается на более раннюю статью (W. F. Eberlein, "The Circular Function(s)", Math. Mag. 9{ix}$, а оттуда синус и косинус.

    III. Вы конкретно спросили, как доказать формулы сложения из определения единичного круга. Мой любимый (но не совсем строгий) метод таков: определение единичного круга сводится к утверждению, что если $R_\theta$ обозначает вращение против часовой стрелки вокруг начала координат на $\theta$ радиан, то $$ R_\theta\left[\begin{matrix} 1 \\ 0 \end{matrix}\right] = \left[\begin{matrix} \cos\theta \\ \sin\theta \end{matrix}\right] $$ С помощью небольшой геометрии вы также можете показать, что $$ R_\theta\left[\begin{matrix} 0 \\ 1 \end{matrix}\right] = \left[\begin{matrix} -\sin\theta \\ \phantom-\cos\theta \end{matrix}\right] $$ Вы можете доказать, что $R_\theta$ линейно (например, потому что это изометрия, фиксирующая начало координат). Итак, у него есть матрица, и два приведенных выше утверждения показывают, что эта матрица $$ \left[\begin{matrix} \cos\theta & -\sin\theta \\ \sin\theta & \phantom-\cos\theta \end{matrix}\right] $$ Теперь интуитивно очевидно, что $R_{a+b}=R_a\circ R_b$; следовательно $$ \left[\begin{matrix} \cos (a+b) & -\sin (a+b) \\ \sin (a+b) & \phantom-\cos (a+b) \end{matrix}\right] знак равно \left[\begin{matrix} \cos a & -\sin a \\ \sin a & \phantom-\cos a \end{matrix}\right] \left[\begin{матрица} \cos b & -\sin b \\ \sin b & \phantom-\cos b \end{matrix}\right] $$ Умножение матриц в RHS и определение соответствующих записей дает вам формулы сложения. [Редактировать: И, конечно же, это в связанном вопросе.] 9\infty$ на $\Bbb R$, и особенно они непрерывны. У вас есть $\cos 0 = 1$ и легко доказать, что $\cos 2<0$, таким образом, $\cos$ имеет корень в $]0, 2[$. Точно так же вы можете доказать, что $\sin x > 0$ в этом интервале, и из $\cos'=-\sin$ вы знаете, что $\cos$ имеет только один корень. Назовите его $\frac{\pi}{2}$ (то есть определите, что $\pi$ равно удвоенному корню). Тогда из формул сложения легко доказать, что $\cos$ и $\sin$ являются $2\pi$-периодическими.

    Теперь вы закончили с основными свойствами этих функций. Если хотите, вы также можете определить $\tan x$ и все другие менее известные функции (секанс, косеканс, котангенс), а также вывести формулы для $\tan (x+y)$ и т.п. И определить обратные триг-функции (ну, строго говоря, они обратны ограничениям триг-функций на тщательно выбранные интервалы, так что они биективны на них) и т. д.

    $\endgroup$

    $\begingroup$

    Насколько я помню из школьного курса математики, для исчисления мы полагались на определение единичного круга, которое согласуется с классическим определением синуса и косинуса для углов в диапазоне $[0, \pi/2]$, если вы принимаете, что мера угла такая же, как длина дуги, на которую он опирается.

    На единичной окружности, пройдя расстояние $2\pi$, вы вернетесь в исходную точку, поэтому вы можете показать, что если $\sin \theta = y$, то $\sin (\theta + 2n\pi) = y$ для любого целого числа $n$. Затем вы можете показать с помощью конгруэнтных треугольников (отраженных поперек оси $x$), что $\sin(-x) = -\sin x$ для любого действительного числа $x$.

    Итак, без ограничения общности, при вычислении $\sin(a - b)$ можно считать, что $-\pi < a - b \le \pi$. Мы можем легко решить случаи $a - b = 0$ и $a - b = \pi$. Кроме того, всякий раз, когда $-\pi < a - b < 0$, мы можем использовать тот факт, что $\sin(a - b) = -\sin(b - a)$, так что на самом деле нам нужно только найдите формулу для $\sin(a - b)$ в случае, когда $0 < a - b < \pi$.

    Пусть $O$ будет центром окружности, пусть $A$ будет точкой, которую вы достигнете, пройдя расстояние $a$ против часовой стрелки по окружности, и пусть $B$ будет точкой, достигнутой после прохождения расстояния $b$. Тогда $\треугольник AOB$ — это треугольник, угол которого при $O$ равен $a - b$. Применим теперь следующее преобразование к координатам $x$ и $y$ точек $A$ и $B$:

    \begin{eqnarray} х \стрелка вправо х \cos b + y \sin b, \\ y \rightarrow y \cos b - x \sin b. \end{eqnarray}

    Пусть $A'$ и $B'$ - точки, координаты которых получены применением этого преобразования к координатам $A$ и $B$ соответственно. Поскольку координаты $B$ равны $(\cos b, \sin b)$, мы можем показать, что координаты $B'$ равны $(1, 0)$. Мы можем использовать формулу Пифагора для пар координат, чтобы доказать, что расстояние от $O = (0,0)$ до $A'$ равно $1$, а расстояние от $A'$ до $B'$ равно расстояние от $A$ до $B$, и теперь мы знаем, что треугольники $\треугольник AOB$ и $\треугольник A'OB'$ конгруэнтны. Следовательно, угол при $O$ в $\треугольнике A'OB'$ имеет меру $a - b$. Между тем, мы можем найти, что $y$-координата $A'$ равна $\sin a \cos b - \cos a \sin b$. Остается только показать, что $\треугольник AOB$ и $\треугольник A'OB'$ имеют одинаковую ориентацию (так что $A'$ находится над осью $x$), что можно сделать, применив формулу к координаты вершин треугольника, которые дают ориентацию треугольника (то есть результат умножается на $-1$, если вы отражаете треугольник), или другие знания об изометрии. Зная это, мы показали, что

    \начало{уравнение} \sin(a - b) = \sin a \cos b - \cos a \sin b. \end{equation}

    Аналогичная процедура работает для формулы разности косинусов, хотя в этом случае нам не нужно доказывать, что $A'$ находится над осью:

    \begin{equation} \cos(a - b) = \cos a \cos b - \sin a \sin b. \end{equation}

    Для $\sin(a + b)$ и $\cos(a + b)$ просто подставьте $-b$ вместо $b$ в уже известных формулах.

    $\endgroup$

    Твой ответ

    Зарегистрируйтесь или войдите в систему

    Зарегистрируйтесь с помощью Google

    Зарегистрироваться через Facebook

    Зарегистрируйтесь, используя электронную почту и пароль

    Опубликовать как гость

    Электронная почта

    Требуется, но никогда не отображается

    Опубликовать как гость

    Электронная почта

    Требуется, но не отображается

    Нажимая «Опубликовать свой ответ», вы соглашаетесь с нашими условиями обслуживания, политикой конфиденциальности и политикой использования файлов cookie

    .

    7.4: Формулы суммы-произведения и произведения-суммы

    1. Последнее обновление
    2. Сохранить как PDF
  • Идентификатор страницы
    1370
    • OpenStax
    • OpenStax
    Цели обучения
    • Экспресс-продукты в сумме.
    • Выразите суммы как продукты.

    Группа марширует по полю, создавая удивительный звук, который поддерживает толпу. Этот звук распространяется как волна, которую можно интерпретировать с помощью тригонометрических функций.

    Рисунок \(\PageIndex{1}\): Марширующий оркестр Калифорнийского университета в Лос-Анджелесе (кредит: Эрик Чан, Flickr).

    Например, рисунок \(\PageIndex{2}\) представляет звуковую волну для музыкальной ноты A. В этом разделе мы исследуем тригонометрические тождества, лежащие в основе таких повседневных явлений, как звуковые волны.

    Рисунок \(\PageIndex{2}\)

    Выражение произведений в виде сумм

    Мы уже изучили ряд формул, полезных для расширения или упрощения тригонометрических выражений, но иногда нам может произведение косинуса и синуса в виде суммы. Мы можем использовать формулы произведения на сумму, которые выражают произведения тригонометрических функций в виде сумм. Давайте сначала исследуем тождество косинуса, а затем тождество синуса.

    Выражение произведений в виде суммы косинуса

    Мы можем вывести формулу произведения на сумму из тождеств суммы и разности для косинуса . Если мы сложим два уравнения, мы получим:

    \[\begin{align*} \cos \alpha \cos \beta+\sin \alpha \sin \beta&= \cos(\alpha-\beta)\\[4pt ] \underline{+ \cos \alpha \cos \beta-\sin \alpha \sin \beta}&= \underline{ \cos(\alpha+\beta) }\\[4pt] 2 \cos \alpha \cos \ beta&= \cos(\alpha-\beta)+\cos(\alpha+\beta)\end{align*}\]

    Затем мы делим на 2, чтобы выделить произведение косинусов:

    \[ \cos \alpha \cos \beta= \dfrac{1}{2}[\cos(\alpha-\beta)+\cos( \alpha+\beta)] \label{eq1}\]

    Как: Произведение косинусов выразить в виде суммы
    1. Запишите формулу произведения косинусов.
    2. Подставить данные углы в формулу.
    3. Упростить.
    Пример \(\PageIndex{1}\): запись произведения в виде суммы с использованием формулы произведения на сумму для косинуса

    Запишите следующее произведение косинусов в виде суммы: \(2\cos\left(\dfrac{7x}{2}\right) \cos\left(\dfrac{3x}{2}\right)\).

    Решение

    Начнем с записи формулы произведения косинусов (уравнение \ref{eq1}):

    \[ \cos \alpha \cos \beta = \dfrac{1}{2}[ \ cos(\alpha-\beta)+\cos(\alpha+\beta) ] \nonumber \]

    Затем мы можем подставить данные углы в формулу и упростить.

    \[\begin{align*} 2 \cos\left(\dfrac{7x}{2}\right)\cos\left(\dfrac{3x}{2}\right)&= 2\left(\ dfrac{1}{2}\right)[ \cos\left(\dfrac{7x}{2}-\dfrac{3x}{2}\right)+\cos\left(\dfrac{7x}{2} +\dfrac{3x}{2}\right) ]\\[4pt] &= \cos\left(\dfrac{4x}{2}\right)+\cos\left(\dfrac{10x}{2} \right) \\[4pt] &= \cos 2x+\cos 5x \end{align*}\]

    Упражнение \(\PageIndex{1}\)

    Используйте формулу произведения на сумму (уравнение \ref{eq1}), чтобы записать произведение в виде суммы или разности: \(\cos(2\theta)\ потому что (4 \ тета) \).

    Ответить

    \(\dfrac{1}{2}(\cos 6\theta+\cos 2\theta)\)

    Выражение произведения синуса и косинуса в виде суммы

    Далее мы выведем формулу произведения на сумму синуса и косинуса из формул суммы и разности для синуса . Если мы добавим тождества суммы и разности, мы получим:

    \[\begin{align*} \cos \alpha \cos \beta+\sin \alpha \sin \beta&= \cos(\alpha-\beta)\\ [4pt] \underline{+ \cos \alpha \cos \beta-\sin \alpha \sin \beta}&= \cos(\alpha+\beta)\\[4pt] 2 \cos \alpha \cos \beta&= \cos(\alpha-\beta)+\cos(\alpha+\beta)\\[4pt] \text{Затем делим на 2, чтобы выделить произведение косинусов:}\\[4pt] \cos \alpha \ cos \beta&= \dfrac{1}{2}\left[\cos(\alpha-\beta)+\cos(\alpha+\beta)\right] \end{align*}\]

    Пример \(\PageIndex{2}\): Запись произведения в виде суммы, содержащей только синус или косинус

    Выразите следующее произведение в виде суммы, содержащей только синус или косинус и не содержащей произведений: \(\sin(4\theta )\cos(2\тета)\).

    Решение

    Напишите формулу произведения синуса и косинуса. Затем подставьте данные значения в формулу и упростите.

    \[\begin{align*} \sin \alpha \cos \beta&= \dfrac{1}{2}[ \sin(\alpha+\beta)+\sin(\alpha-\beta) ]\\[ 4pt] \sin(4\theta)\cos(2\theta)&= \dfrac{1}{2}[\sin(4\theta+2\theta)+\sin(4\theta-2\theta) ]\\[4pt] &= \dfrac{1}{2}[\sin(6\theta)+\sin(2\theta)] \end{align*}\]

    Упражнение \(\PageIndex{2}\)

    Используйте формулу произведения на сумму, чтобы записать произведение в виде суммы: \(\sin(x+y)\cos(x−y)\).

    Ответить

    \(\dfrac{1}{2}(\sin 2x+\sin 2y)\)

    Выражение произведения синусов через косинус

    Выражение произведения синусов через косинус также получается из тождеств суммы и разности для косинуса. В этом случае мы сначала вычтем две формулы косинуса:

    \[\begin{align*} \cos(\alpha-\beta)&= \cos \alpha \cos \beta+\sin \alpha \sin \beta\\[4pt] \underline{-\cos(\ alpha+\beta)}&= -(\cos\alpha\cos\beta-\sin\alpha\sin\beta)\\[4pt] \cos(\alpha-\beta)-\cos(\alpha+\beta) &= 2 \sin \alpha \sin \beta\\[4pt] \text{Затем делим на 2, чтобы выделить произведение синусов:}\\[4pt] \sin \alpha \sin \beta&= \dfrac{ 1}{2}[ \cos(\alpha-\beta)-\cos(\alpha+\beta) ] \end{align*}\]

    Точно так же мы могли бы выразить произведение косинусов через синус или вывести другое формулы произведения на сумму.

    ФОРМУЛЫ ПРОИЗВЕДЕНИЯ НА СУММУ

    Формулы произведения на сумму следующие:

    \[\cos \alpha \cos \beta=\dfrac{1}{2}[\cos(\ alpha-\beta)+\cos(\alpha+\beta)]\]

    \[\sin \alpha \cos \beta=\dfrac{1}{2}[\sin(\alpha+\beta)+\sin (\alpha-\beta)]\]

    \[\sin \alpha \sin \beta=\dfrac{1}{2}[\cos(\alpha-\beta)-\cos(\alpha+\beta) ]\]

    \[\cos \alpha \sin \beta=\dfrac{1}{2}[\sin(\alpha+\beta)−\sin(\alpha−\beta)]\]

    Пример \ (\PageIndex{3}\): Выразите произведение в виде суммы или разности

    Запишите \(\cos(3\theta) \cos(5\theta)\) в виде суммы или разности.

    Решение

    У нас есть произведение косинусов, поэтому начнем с написания соответствующей формулы. Затем подставляем данные углы и упрощаем.

    \[\begin{align*} \cos \alpha \cos \beta&= \dfrac{1}{2}[\cos(\alpha-\beta)+\cos(\alpha+\beta)]\\[ 4pt] \cos(3\theta)\cos(5\theta)&= \dfrac{1}{2}[\cos(3\theta-5\theta)+\cos(3\theta+5\theta) ]\\[4pt] &= \dfrac{1}{2}[\cos(2\theta)+\cos(8\theta)]\qquad \text{Использовать четно-нечетное тождество} \end{align*} \]

    Упражнение \(\PageIndex{3}\)

    Используйте формулу произведения на сумму для вычисления \(\cos \dfrac{11\pi}{12} \cos \dfrac{\pi}{12}\ ).

    Ответить

    \(\dfrac{−2−\sqrt{3}}{4}\)

    Выражение сумм как произведений

    Некоторые задачи требуют обратного процесса, который мы только что использовали. Формулы суммы к произведению позволяют нам выразить суммы синуса или косинуса в виде произведений. Эти формулы могут быть получены из тождеств произведения на сумму. Например, с помощью нескольких замен мы можем получить тождество суммы и произведения для синус . Пусть \(\dfrac{u+v}{2}=\alpha\) и \(\dfrac{u−v}{2}=\beta\).

    Затем

    \[\begin{align*} \alpha+\beta&= \dfrac{u+v}{2}+\dfrac{u-v}{2}\\[4pt] &= \dfrac{2u} {2}\\[4pt] &= u \end{align*}\]

    \[\begin{align*} \alpha-\beta&= \dfrac{u+v}{2}-\dfrac{u-v }{2}\\[4pt] &= \dfrac{2v}{2}\\[4pt] &= v \end{align*}\]

    Таким образом, замена \(\alpha\) и \(\ бета\) в формуле произведения на сумму с подстановочными выражениями имеем

    \[\begin{align*} \sin \alpha \cos \beta&= \dfrac{1}{2}[\sin(\alpha+\beta)+\sin(\alpha-\beta)]\\[4pt ] \sin \left ( \frac{u+v}{2} \right ) \cos \left ( \frac{u-v}{2} \right )&= \frac{1}{2}[\sin u + \sin v]\qquad \text{Замените} (\alpha+\beta) \text{ и } (\alpha\beta)\\[4pt] 2\sin\left(\dfrac{u+v}{2} \right) \cos\left(\dfrac{u-v}{2}\right)&= \sin u+\sin v \end{align*}\]

    Другие тождества суммы-произведения выводятся аналогично.

    ФОРМУЛЫ СУММЫ-ПРОИЗВЕДЕНИЕ

    Формулы суммы-произведения следующие:

    \[\sin \alpha+\sin \beta=2\sin\left(\dfrac{\alpha+\beta}{2}\right)\cos \left(\dfrac{\alpha-\beta}{2}\right)\]

    \[\sin\alpha-\sin\beta=2\sin\left(\dfrac{\alpha-\beta}{ 2}\right)\cos\left(\dfrac{\alpha+\beta}{2}\right)\]

    \[\cos \alpha-\cos \beta=-2\sin\left(\dfrac{ \alpha+\beta}{2}\right)\sin\left(\dfrac{\alpha-\beta}{2}\right)\]

    \[\cos \alpha+\cos \beta=2\sin\ влево(\dfrac{\alpha+\beta}{2}\right)\sin\left(\dfrac{\alpha-\beta}{2}\right)\]

    Пример \(\PageIndex{4}\): Запись разности синусов в виде произведения

    Запишите следующее выражение разности синусов в виде произведения: \(\sin(4\theta)−\sin(2\theta) \).

    Решение

    Начнем с написания формулы разности синусов.

    \[\begin{align*} \sin \alpha-\sin \beta&= 2\sin\left(\dfrac{\alpha-\beta}{2}\right)\cos\left(\dfrac{\ alpha+\beta}{2}\right)\\[4pt] \text {Подставьте значения в формулу и упростите. }\\[4pt] \sin(4\theta)-\sin(2\theta)& = 2\sin\left(\dfrac{4\theta-2\theta}{2}\right) \cos\left(\dfrac{4\theta+2\theta}{2}\right)\\[4pt ] &= 2\sin\left(\dfrac{2\theta}{2}\right) \cos\left(\dfrac{6\theta}{2}\right)\\[4pt] &= 2 \sin \тета \cos(3\тета) \end{выравнивание*}\]

    Упражнение \(\PageIndex{4}\)

    Используйте формулу приведения суммы к произведению, чтобы записать сумму в виде произведения: \(\sin(3\theta)+\sin(\theta)\).

    Ответить

    \(2\sin(2\тета)\cos(\тета)\)

    Пример \(\PageIndex{5}\): вычисление с использованием формулы приведения суммы к произведению

    Вычисление \(\cos(15°)−\cos(75°)\). Проверьте ответ с помощью графического калькулятора.

    Решение

    Начнем с написания формулы разности косинусов. 9{\ circ}) \\ [4pt]
    & = -2 \ влево (\ dfrac {\ sqrt {2}} {2} \ вправо) \ влево (- \ dfrac {1} {2} \ вправо) \\ [4pt]
    &= \dfrac{\sqrt{2}}{2}
    \end{align*}\]

    Пример \(\PageIndex{6}\): Подтверждение личности

    Подтверждение личности:

    \[\dfrac{\cos(4t)−\cos(2t)}{\sin(4t)+\sin(2t)}=−\tan t\]

    Решение

    Начнем с левую часть, более сложную часть уравнения, и переписать выражение, пока оно не совпадет с правой частью.

    \[\begin{align*} \dfrac{\cos(4t)-\cos(2t)}{\sin(4t)+\sin(2t)}&= \dfrac{-2 \sin\left( \dfrac{4t+2t}{2}\right) \sin\left(\dfrac{4t-2t}{2}\right)}{2 \sin\left(\dfrac{4t+2t}{2}\ справа) \cos\left(\dfrac{4t-2t}{2}\right)}\\[4pt] &= \dfrac{-2 \sin(3t)\sin t}{2 \sin(3t)\ cos t}\\[4pt] &= -\dfrac{\sin t}{\cos t}\\[4pt] &= -\tan t \end{align*}\]

    Анализ

    Отзыв что проверка тригонометрических тождеств имеет свой собственный набор правил. Процедуры решения уравнения не совпадают с процедурами проверки личности. Когда мы подтверждаем тождество, мы выбираем одну сторону для работы и делаем замены до тех пор, пока эта сторона не превратится в другую сторону. 92 \ тета \ конец {выравнивание *} \]

    Средства массовой информации

    Получите доступ к этим онлайн-ресурсам для получения дополнительных инструкций и практических занятий с идентификаторами продуктов и сумм.

    • Сумма идентификаторов продуктов
    • Сумма к продукту и продукт к сумме идентификаторов

    Ключевые уравнения

    Формулы произведения на сумму

    \[\cos \alpha \cos \beta=\dfrac{1}{2}[\cos(\alpha-\beta)+\cos(\ альфа+\бета)] \номер \]

    \[\sin \alpha \cos \beta=\dfrac{1}{2}[\sin(\alpha+\beta)+\sin(\alpha-\beta)] \nonumber \]

    \[\sin \alpha \sin \beta=\dfrac{1}{2}[\cos(\alpha-\beta)−\cos(\alpha+\beta)] \nonumber \]

    \[\cos \alpha \sin \ beta=\dfrac{1}{2}[\sin(\alpha+\beta)−\sin(\alpha−\beta)] \nonumber \]

    Формулы суммы к произведению

    \[\sin \alpha+\sin \beta=2\sin(\dfrac{\alpha+\beta}{2})\cos(\dfrac{\alpha-\beta}{2}) \nonumber \]

    \[\sin \ альфа-\sin\beta=2\sin(\dfrac{\alpha-\beta}{2})\cos(\dfrac{\alpha+\beta}{2}) \nonumber \]

    \[\cos \alpha-\cos \beta=-2\sin(\dfrac{\alpha+\beta}{2})\sin(\dfrac{\alpha-\beta}{2}) \nonumber \ ]

    \[\cos \alpha+\cos \beta=2\sin(\dfrac{\alpha+\beta}{2})\sin(\dfrac{\alpha-\beta}{2}) \nonumber \]

    Ключевые понятия

    • Из тождеств суммы и разности мы можем вывести формулы произведения на сумму и формулы произведения суммы на синус и косинус.
    • Мы можем использовать формулы произведения на сумму, чтобы переписать произведения синусов, произведения косинусов и произведения синусов и косинусов в виде сумм или разностей синусов и косинусов. См. Пример \(\PageIndex{1}\), Пример \(\PageIndex{2}\) и Пример \(\PageIndex{3}\).
    • Мы также можем получить тождества суммы-произведения из тождеств произведения-суммы, используя подстановку.
    • Мы можем использовать формулы преобразования суммы в произведение, чтобы переписать сумму или разность синусов, косинусов или произведений синуса и косинуса как произведения синусов и косинусов. См. пример \(\PageIndex{4}\).
    • Тригонометрические выражения часто проще вычислить с помощью формул. См. пример \(\PageIndex{5}\).
    • Тождества можно проверить с помощью других формул или преобразования выражений в синусы и косинусы. Для проверки тождества мы выбираем более сложную сторону знака равенства и переписываем ее до тех пор, пока она не преобразуется в другую сторону. См. Пример \(\PageIndex{6}\) и Пример \(\PageIndex{7}\).

    Эта страница под заголовком 7.4: Формулы суммы к продукту и продукту к сумме распространяется под лицензией CC BY 4.0 и была создана, изменена и/или курирована OpenStax с использованием исходного контента, который был отредактирован в соответствии со стилем и стандартами. платформы LibreTexts; подробная история редактирования доступна по запросу.

    1. Наверх
      • Была ли эта статья полезной?
      1. Тип изделия
        Раздел или страница
        Автор
        ОпенСтакс
        Лицензия
        СС BY
        Версия лицензии
        4,0
        Программа ООР или издатель
        ОпенСтакс
        Показать страницу TOC
        нет
        Включено
        да
      2. Теги
        1. Произведение на формулы суммирования
        2. источник@https://openstax. org/details/books/precalculus
        3. Суммирование формул произведения

      Закон косинусов

      Горячая математика

      Закон косинусов используется для нахождения оставшихся частей косой (неправильной) треугольник когда известны либо длины двух сторон и мера прилежащего угла (SAS), либо известны длины трех сторон (SSS). Ни в одном из этих случаев невозможно использовать Закон синусов потому что мы не можем установить разрешимую пропорцию.

      Закон косинусов гласит:

      с 2 знак равно а 2 + б 2 − 2 а б потому что С .

      Это похоже на Теорема Пифагора за исключением третьего срока и если С угол прямой третий член равен 0 потому что косинус 90 ° является 0 и получаем теорему Пифагора. Итак, теорема Пифагора является частным случаем закона косинусов.

      Закон косинусов можно также сформулировать как

      б 2 знак равно а 2 + с 2 − 2 а с потому что Б или же

      а 2 знак равно б 2 + с 2 − 2 б с потому что А .

      Пример 1: Две стороны и прилежащий угол-SAS

      Данный а знак равно 11 , б знак равно 5 а также м ∠ С знак равно 20 ° . Найдите оставшуюся сторону и углы.

      с 2 знак равно а 2 + б 2 − 2 а б потому что С

      с знак равно а 2 + б 2 − 2 а б потому что С

      знак равно 11 2 + 5 2 − 2 ( 11 ) ( 5 ) ( потому что 20 ° )

      ≈ 6,53

      Чтобы найти оставшиеся углы, проще всего теперь использовать закон синусов.

      грех А ≈ 11 грех 20 ° 6,53

      А ≈ 144,82 °

      грех Б ≈ 5 грех 20 ° 6,53

      Б ≈ 15.2 °

      Обратите внимание, что угол А противоположна наибольшей стороне, и треугольник не является прямоугольным. Итак, когда вы берете обратную, вам нужно учитывать тупой угол, синус которого равен 11 грех ( 20 ° ) 6,53 ≈ 0,5761 .

      Пример 2: Три стороны-SSS

      Данный а знак равно 8 , б знак равно 19 а также с знак равно 14 . Найдите меры углов.

      Лучше всего сначала найти угол, противоположный большей стороне. В данном случае это сторона б .

      потому что Б знак равно б 2 − а 2 − с 2 − 2 а с знак равно 192 − 8 2 − 14 2 − 2 ( 8 ) ( 14 ) ≈ − 0,45089

      С потому что Б отрицательно, мы знаем, что Б является тупым углом.

      Б ≈ 116,80 °

      С Б тупой угол, а треугольник имеет не более одного тупого угла, мы знаем, что угол А и угол С оба острые.

      Чтобы найти два других угла, проще всего использовать закон синусов.

      а грех А знак равно б грех Б знак равно с грех С

      8 грех А ≈ 19грех 116,80 ° ≈ 14 грех С

      грех А ≈ 8 грех 116,80 ° 19

      А ≈ 22.

      Добавить комментарий

      Ваш адрес email не будет опубликован. Обязательные поля помечены *